Vous êtes sur la page 1sur 196

6 th

Edition

GMAT Sets & Statistics Guide


Joern Meissner

TURBOCHARGE
GMAT and GMAT CAT are registered trademarks
YOUR PREP
of the Graduate Management Admission Council (GMAC).
GMAC does not endorse nor is it affiliated in any way with
the owner of this product or any content herein.

+1 (212) 316-2000 www.manhattanreview.com


Turbocharge Your GMAT:
Sets & Statistics Guide
part of the 6th Edition Series
April 20th, 2016

 Complete & Challenging Training Set


Problem Solving - 60 Questions
Data Sufficiency - 40 Questions

 Questions mapped according to the scope


of the GMAT
 Ample questions on Data Interpretation
 Excellent collection of DS questions on
Statistics
 Text-cum-graphic explanations

www.manhattanreview.com
19992016 Manhattan Review. All Rights Reserved.
ii Sets & Statistics Guide

Copyright and Terms of Use

Copyright and Trademark


All materials herein (including names, terms, trademarks, designs, images, and graphics) are
the property of Manhattan Review, except where otherwise noted. Except as permitted herein,
no such material may be copied, reproduced, displayed or transmitted or otherwise used with-
out the prior written permission of Manhattan Review. You are permitted to use material
herein for your personal, noncommercial use, provided that you do not combine such material
into a combination, collection, or compilation of material. If you have any questions regarding
the use of the material, please contact Manhattan Review at info@manhattanreview.com.

This material may make reference to countries and persons. The use of such references is for
hypothetical and demonstrative purposes only.

Terms of Use
By using this material, you acknowledge and agree to the terms of use contained herein.

No Warranties
This material is provided without warranty, either express or implied, including the implied
warranties of merchantability, of fitness for a particular purpose and noninfringement. Man-
hattan Review does not warrant or make any representations regarding the use, accuracy or
results of the use of this material. This material may make reference to other source materials.
Manhattan Review is not responsible in any respect for the content of such other source ma-
terials, and disclaims all warranties and liabilities with respect to the other source materials.

Limitation on Liability
Manhattan Review shall not be responsible under any circumstances for any direct, indirect,
special, punitive, or consequential damages (Damages) that may arise from the use of this
material. In addition, Manhattan Review does not guarantee the accuracy or completeness of
its course materials, which are provided as is with no warranty, express or implied. Man-
hattan Review assumes no liability for any Damages from errors or omissions in the material,
whether arising in contract, tort or otherwise.

GMAT is a registered trademark of the Graduate Management Admission Council.


GMAC does not endorse, nor is it affiliated in any way with, the owner of this product or any
content herein.
10-Digit International Standard Book Number: (ISBN: 1-62926-063-0)
13-Digit International Standard Book Number: (ISBN: 978-1-62926-063-1)

Last updated on April 20th, 2016.

Manhattan Review, 275 Madison Avenue, Suite 1429, New York, NY 10016.
Phone: +1 (212) 316-2000. E-Mail: info@manhattanreview.com. Web: www.manhattanreview.com

www.manhattanreview.com 19992016 Manhattan Review


Sets & Statistics Guide iii

About the Turbocharge your GMAT Series

The Turbocharge Your GMAT Series is carefully designed to be clear, comprehensive, and
content-driven. Long regarded as the gold standard in GMAT prep worldwide, Manhattan Re-
views GMAT prep books offer professional GMAT instruction for dramatic score improvement.
Now in its updated 6th edition, the full series is designed to provide GMAT test-takers with
complete guidance for highly successful outcomes. As many students have discovered, Man-
hattan Reviews GMAT books break down the different test sections in a coherent, concise,
and accessible manner. We delve deeply into the content of every single testing area and zero
in on exactly what you need to know to raise your score. The full series is comprised of 16
guides that cover concepts in mathematics and grammar from the most basic through the
most advanced levels, making them a great study resource for all stages of GMAT preparation.
Students who work through all of our books benefit from a substantial boost to their GMAT
knowledge and develop a thorough and strategic approach to taking the GMAT.

 GMAT Math Essentials (ISBN: 978-1-62926-057-0)


 GMAT Number Properties Guide (ISBN: 978-1-62926-058-7)
 GMAT Arithmetic Guide (ISBN: 978-1-62926-059-4)
 GMAT Algebra Guide (ISBN: 978-1-62926-060-0)
 GMAT Geometry Guide (ISBN: 978-1-62926-061-7)
 GMAT Word Problems Guide (ISBN: 978-1-62926-062-4)
 GMAT Sets & Statistics Guide (ISBN: 978-1-62926-063-1)
 GMAT Combinatorics & Probability Guide (ISBN: 978-1-62926-064-8)
 GMAT Data Sufficiency Guide (ISBN: 978-1-62926-065-5)
 GMAT Quantitative Question Bank (ISBN: 978-1-62926-066-2)
 GMAT Sentence Correction Guide (ISBN: 978-1-62926-067-9)
 GMAT Critical Reasoning Guide (ISBN: 978-1-62926-068-6)
 GMAT Reading Comprehension Guide (ISBN: 978-1-62926-069-3)
 GMAT Integrated Reasoning Guide (ISBN: 978-1-62926-070-9)
 GMAT Analytical Writing Guide (ISBN: 978-1-62926-071-6)
 GMAT Vocabulary Builder (ISBN: 978-1-62926-072-3)

19992016 Manhattan Review www.manhattanreview.com


iv Sets & Statistics Guide

About the Company


Manhattan Reviews origin can be traced directly back to an Ivy League MBA classroom in 1999.
While teaching advanced quantitative subjects to MBAs at Columbia Business School in New
York City, Professor Dr. Joern Meissner developed a reputation for explaining complicated
concepts in an understandable way. Remembering their own less-than-optimal experiences
preparing for the GMAT, Prof. Meissners students challenged him to assist their friends, who
were frustrated with conventional GMAT preparation options. In response, Prof. Meissner
created original lectures that focused on presenting GMAT content in a simplified and intel-
ligible manner, a method vastly different from the voluminous memorization and so-called
tricks commonly offered by others. The new approach immediately proved highly popular
with GMAT students, inspiring the birth of Manhattan Review.

Since its founding, Manhattan Review has grown into a multi-national educational services
firm, focusing on GMAT preparation, MBA admissions consulting, and application advisory
services, with thousands of highly satisfied students all over the world. The original lectures
have been continuously expanded and updated by the Manhattan Review team, an enthusiastic
group of master GMAT professionals and senior academics. Our team ensures that Manhattan
Review offers the most time-efficient and cost-effective preparation available for the GMAT.
Please visit www.ManhattanReview.com for further details.

About the Founder


Professor Dr. Joern Meissner has more than 25 years of teaching experience at the graduate
and undergraduate levels. He is the founder of Manhattan Review, a worldwide leader in test
prep services, and he created the original lectures for its first GMAT preparation class. Prof.
Meissner is a graduate of Columbia Business School in New York City, where he received a PhD
in Management Science. He has since served on the faculties of prestigious business schools
in the United Kingdom and Germany. He is a recognized authority in the areas of supply chain
management, logistics, and pricing strategy. Prof. Meissner thoroughly enjoys his research,
but he believes that grasping an idea is only half of the fun. Conveying knowledge to others
is even more fulfilling. This philosophy was crucial to the establishment of Manhattan Review,
and remains its most cherished principle.

www.manhattanreview.com 19992016 Manhattan Review


Sets & Statistics Guide v

The Advantages of Using Manhattan Review


I Time efficiency and cost effectiveness.
For most people, the most limiting factor of test preparation is time.
It takes significantly more teaching experience to prepare a student in less time.
Our test preparation approach is tailored for busy professionals. We will teach you
what you need to know in the least amount of time.

I Our high-quality and dedicated instructors are committed to helping every student
reach her/his goals.

19992016 Manhattan Review www.manhattanreview.com


vi Sets & Statistics Guide

International Phone Numbers and Official Manhattan Review Websites


Manhattan Headquarters +1-212-316-2000 www.manhattanreview.com
USA & Canada +1-800-246-4600 www.manhattanreview.com
Argentina +1-212-316-2000 www.review.com.ar
Australia +61-3-9001-6618 www.manhattanreview.com
Austria +43-720-115-549 www.review.at
Belgium +32-2-808-5163 www.manhattanreview.be
Brazil +1-212-316-2000 www.manhattanreview.com.br
Chile +1-212-316-2000 www.manhattanreview.cl
China +86-20-2910-1913 www.manhattanreview.cn
Czech Republic +1-212-316-2000 www.review.cz
France +33-1-8488-4204 www.review.fr
Germany +49-89-3803-8856 www.review.de
Greece +1-212-316-2000 www.review.com.gr
Hong Kong +852-5808-2704 www.review.hk
Hungary +1-212-316-2000 www.review.co.hu
India +1-212-316-2000 www.review.in
Indonesia +1-212-316-2000 www.manhattanreview.id
Ireland +1-212-316-2000 www.gmat.ie
Italy +39-06-9338-7617 www.manhattanreview.it
Japan +81-3-4589-5125 www.manhattanreview.jp
Malaysia +1-212-316-2000 www.review.my
Mexico +1-212-316-2000 www.manhattanreview.mx
Netherlands +31-20-808-4399 www.manhattanreview.nl
New Zealand +1-212-316-2000 www.review.co.nz
Philippines +1-212-316-2000 www.review.ph
Poland +1-212-316-2000 www.review.pl
Portugal +1-212-316-2000 www.review.pt
Qatar +1-212-316-2000 www.review.qa
Russia +1-212-316-2000 www.manhattanreview.ru
Singapore +65-3158-2571 www.gmat.sg
South Africa +1-212-316-2000 www.manhattanreview.co.za
South Korea +1-212-316-2000 www.manhattanreview.kr
Sweden +1-212-316-2000 www.gmat.se
Spain +34-911-876-504 www.review.es
Switzerland +41-435-080-991 www.review.ch
Taiwan +1-212-316-2000 www.gmat.tw
Thailand +66-6-0003-5529 www.manhattanreview.com
Turkey +1-212-316-2000 www.review.com.tr
United Arab Emirates +1-212-316-2000 www.manhattanreview.ae
United Kingdom +44-20-7060-9800 www.manhattanreview.co.uk
Rest of World +1-212-316-2000 www.manhattanreview.com

www.manhattanreview.com 19992016 Manhattan Review


Contents

1 Welcome 1

2 Concepts 3
2.1 Sets . . . . . . . . . . . . . . . . . . . . . . . . . . . . . . . . . . . . . . . . . . . . . . . . . 4
2.2 Statistics . . . . . . . . . . . . . . . . . . . . . . . . . . . . . . . . . . . . . . . . . . . . . 10
2.3 Data Interpretation . . . . . . . . . . . . . . . . . . . . . . . . . . . . . . . . . . . . . . . 16

3 Questions 31
3.1 Problem Solving Questions . . . . . . . . . . . . . . . . . . . . . . . . . . . . . . . . . . . 32
3.1.1 Sets . . . . . . . . . . . . . . . . . . . . . . . . . . . . . . . . . . . . . . . . . . . . . 32
3.1.2 Statistics . . . . . . . . . . . . . . . . . . . . . . . . . . . . . . . . . . . . . . . . . . 37
3.1.3 Data Interpretation . . . . . . . . . . . . . . . . . . . . . . . . . . . . . . . . . . . 56
3.2 Data Sufficiency Questions . . . . . . . . . . . . . . . . . . . . . . . . . . . . . . . . . . . 66
3.2.1 Sets . . . . . . . . . . . . . . . . . . . . . . . . . . . . . . . . . . . . . . . . . . . . . 67
3.2.2 Statistics . . . . . . . . . . . . . . . . . . . . . . . . . . . . . . . . . . . . . . . . . . 68
3.2.3 Data Interpretation . . . . . . . . . . . . . . . . . . . . . . . . . . . . . . . . . . . 79

4 Answer Key 83
4.1 Problem Solving Questions . . . . . . . . . . . . . . . . . . . . . . . . . . . . . . . . . . . 84
4.2 Data Sufficiency Questions . . . . . . . . . . . . . . . . . . . . . . . . . . . . . . . . . . . 85

5 Solution 87
5.1 Problem Solving Questions . . . . . . . . . . . . . . . . . . . . . . . . . . . . . . . . . . . 88
5.1.1 Sets . . . . . . . . . . . . . . . . . . . . . . . . . . . . . . . . . . . . . . . . . . . . . 88
5.1.2 Statistics . . . . . . . . . . . . . . . . . . . . . . . . . . . . . . . . . . . . . . . . . . 98
5.1.3 Data Interpretation . . . . . . . . . . . . . . . . . . . . . . . . . . . . . . . . . . . 129
5.2 Data Sufficiency Questions . . . . . . . . . . . . . . . . . . . . . . . . . . . . . . . . . . . 141
5.2.1 Sets . . . . . . . . . . . . . . . . . . . . . . . . . . . . . . . . . . . . . . . . . . . . . 142
5.2.2 Statistics . . . . . . . . . . . . . . . . . . . . . . . . . . . . . . . . . . . . . . . . . . 145
5.2.3 Data Interpretation . . . . . . . . . . . . . . . . . . . . . . . . . . . . . . . . . . . 177

6 Talk to Us 183

vii
viii Sets & Statistics Guide

www.manhattanreview.com 19992016 Manhattan Review


Chapter 1

Welcome

Dear Students,
Here at Manhattan Review, we constantly strive to provide you the best educational content
for standardized test preparation. We make a tremendous effort to keep making things better
and better for you. This is especially important with respect to an examination such as the
GMAT. A typical GMAT aspirant is confused with so many test-prep options available. Your
challenge is to choose a book or a tutor that prepares you for attaining your goal. We cannot
say that we are one of the best, it is you who has to be the judge.

There are umpteen numbers of books on Sets, Statistics and Data Interpretation for GMAT
preparation. What is so different about this book? The answer lies in its approach to deal with
the questions. Solution of each question is dealt with in detail. There are many questions that
have been solved through alternate approaches. The objective is to understand questions from
multiple aspects. Few seemingly scary questions have been solved through Logical Deduction
or through Intuitive approach.

The book has a great collection of 100 GMAT-like questions: 60 PS and 40 DS.

Apart from books on Number Properties, Word Problem, Algebra, Arithmetic, Geometry,
Permutation and Combination, and Sets and Statistics which are solely dedicated to GMAT-
QA-PS & DS, the book on GMAT-Math Essentials is solely dedicated to develop your math
fundamentals. Another publication GMAT Quantitative Ability Question Bank boasts of a col-
lection of 500 GMAT like questions.

The Manhattan Reviews GMAT-Sets and Statistics book is holistic and comprehensive in
all respects. Should you have any comments or questions, please feel free to write to us at
info@manhattanreview.com.

Happy Learning!

Professor Dr. Joern Meissner


& The Manhattan Review Team

1
2 Sets & Statistics Guide Concepts

www.manhattanreview.com 19992016 Manhattan Review


Chapter 2

Concepts

3
4 Sets & Statistics Guide Concepts

2.1 Sets
A Set is a collection of well-defined things. The objects of a set are called its elements or mem-
bers.

Nomenclature:

Universal Set: All sets are assumed to be contained in a set called the universal set.

Null Set: An empty set having no elements in it is called a null set.

Disjoint Set: Two sets are disjoint if they have no elements in common.
For example: A = {1, 2, 3} and B = {4, 5, 6} are two disjoint sets.

Cardinal Number of a Set: The Cardinal number of a finite set A is the number of ele-
ments of the set, denoted by n (A). For example: For A = {1, 2, 3} , n (A) = 3

Venn Diagrams: A Venn diagram is a pictorial representation of sets represented by enclosed


areas in a plane. The universal set is represented by a rectangle, and the other sets are repre-
sented by areas lying within it.

Set Operations:

Union: The Union of two sets A and B, i.e. A B, is a set that contains all the elements
contained in set A or set B.

For example: If A = {2, 3} , B = {1, 3, 5}

=> A B = {1, 2, 3, 5}

Intersection: Intersection of two sets A and B, i.e. A B, that contains the elements
common to both sets A and B.

For example: For A = {2, 3} , B = {1, 3, 5} , A B = {3}

www.manhattanreview.com 19992016 Manhattan Review


Sets & Statistics Guide Concepts 5

Difference of Two Sets: Difference of two sets A and B, i.e. A B is a set of elements
present in set A but not in set B.

A B

For example: For A = {2, 4} , B = {2, 6, 5} , A B = {4} , B A = {5, 6}

Complement: Complement of set A, i.e. A1 or A0 is a set that contains the elements


outside set A.

For example: If A = {1, 2} and Universal Set U = {1, 2, 7, 8} , A0 = {7, 8}

A few important rules:

For two sets A and B:

A B = A + B A B . . . (i)
(A B)0 = A0 B 0 . . . (ii)
(A B)0 = A0 B 0 . . . (iii)

Let us see how:

We refer to the diagram given below:


Universal Set

The cardinal numbers of the regions are shown as p, q, r and s.

19992016 Manhattan Review www.manhattanreview.com


6 Sets & Statistics Guide Concepts

We have:

A = p + r, B = q + r, A B = r, A B = p + r + q

Thus, we have:

 
AB =p+q+r = p+r + q+r r

= A + B A B . . . (i)

=> (A B)0 = s

We also have: A0 = q + s, B 0 = p + s => A0 B 0 = s

=> (A B)0 = A0 B 0 . . . (ii)

Similarly:

(A B)0 = p + q + s

Also, A0 B 0 = A0 + B 0 A0 B 0 = q + s + p + s s = p + q + s
 

=> (A B)0 = A0 B 0 . . . (iii)

For three sets A, B and C:

A B C = A + B + C (A B + B C + C A) + A B C

Let us see how:

The cardinal numbers of the regions are shown:

Universal Set

www.manhattanreview.com 19992016 Manhattan Review


Sets & Statistics Guide Concepts 7

A=p+s+t+v

B =q+s+u+v

C =r +t+u+v

A B = s + v, A C = t + v, B C = u + v

ABC =v

ABC =p+q+r +s+t+u+v

Also, A B + B C + C A = (s + v) + (t + v) + (u + v) = s + t + u + 3v

=> A + B + C (A B + B C + C A) + A B C

 
= p + s + t + v + q + s + u + v + (r + t + u + v) (s + t + u + 3v) + v

=p+q+r +s+t+u+v

=ABC

Let us take a few examples:

(1) In a survey, it was found that 65 people keep dogs as pets, 70 people keep cats, 40 people
keep birds, 20 people keep both dogs and cats, 10 people keep both cats and birds, 5
people keep both birds and dogs and 3 people keep all three. How many people partici-
pated in the survey if it is known that each person has at least one pet?

Explanation:

Dogs ()
65

17 2
3

7
70 40
Cats () Birds ()

A = 65, B = 40, C = 70

19992016 Manhattan Review www.manhattanreview.com


8 Sets & Statistics Guide Concepts

A B = 20, B C = 10, C A = 5

ABC =3

The total number of people surveyed

=ABC

= A + B + C (A B + B C + C A) + A B C

= 65 + 40 + 70 (20 + 10 + 5) + 3 = 143

(2) In a class of 50 boys, some play cricket, some football and some hockey. The number
of boys who play cricket is more than those who play football, which is more than those
who play hockey, which is more than those who play only two games, which is more
than those who play all three games. If each boy plays at least one game and there is at
least one boy who plays all three games, what is the minimum number of boys who play
football?

Explanation:

We know that the boys play cricket (C), football (F) or hockey (H).

As the information,

50 = C + F + H Only two games + All three games

In order to minimize F, we need to minimize the number of students in the regions a, b,


c and d, as shown in the diagram below.

Cricket (C) Football (F)

c b

Hockey (H)
50 = C + F + H (a + b + c) + d

Since C > F > H > (a + b + c) > d 1 , we assume d = 1, a + b + c = 2, and H = 3.

www.manhattanreview.com 19992016 Manhattan Review


Sets & Statistics Guide Concepts 9

This follows that F = 4 (minimum)

So, we make the following distribution:

Cricket (C) Football (F)

a = 1
44 2

d = 1

c = 1 b = 0

Hockey (H)

We have: C = 50 4 3 2 1 = 44, F = 4, H = 3, a = 1, b = 0, c = 1, and d = 1.

19992016 Manhattan Review www.manhattanreview.com


10 Sets & Statistics Guide Concepts

2.2 Statistics
Statistics is the study of the collection, analysis and interpretation of data. Some of the most
commonly used statistical measures are the Mean, Median, Range, and Standard Deviation.

Let us discuss them.

Arithmetic mean:

For a set of n numbers x1 , x2 , x3 , . . . xn , the arithmetic mean is calculated as:

Sum of the n numbers x1 + x2 + x3 + + xn


x= =
n n

If x1 , x2 , x3 , . . . xn be n observations and f1 , f2 , f3 , . . . fn , be their corresponding


frequencies, their arithmetic mean is calculated as:

x1 f1 + x2 f2 + . . . xn fn
x=
f1 + f2 + + fn

For ease of calculations, the mean can be calculated easily using the method of deviation about
the mean, as shown below:

Let the assumed mean be xa

The observations are modified by subtracting xa from each of the values as follows:

x1 x1 xa

x2 x2 xa

...

...

xn xn xa

The means are now calculated as shown:

For a set of n numbers x1 , x2 , x3 , . . . xn , the arithmetic mean is calculated as:

(x1 xa ) + (x2 xa ) + (x3 xa ) + + (xn xa )


x = xa +
n

If x1 , x2 , x3 , . . . xn be n observations and f1 , f2 , f3 , . . . fn , be their corresponding


frequencies, their arithmetic mean is calculated as:

www.manhattanreview.com 19992016 Manhattan Review


Sets & Statistics Guide Concepts 11

(x1 xa ) f1 + (x2 xa ) f2 + . . . (xn xa ) fn


x = xa +
f1 + f2 + + fn

Let us take a few examples:

(1) What is the mean of 123, 134, 128, 139, 141, and 126?

Explanation:

123 + 134 + 128 + 139 + 141 + 127


Mean = = 132
6

Alternate Approach:

Let the assumed mean = 134.

Thus, the deviations are:

123 134 = 11
134 134 = 0
128 134 = 6
139 134 = 5
141 134 = 7
127 134 = 7

11 + 0 6 + 5 + 7 7 12
=>Mean = 134 + = 134 = 132
6 6

(2) What is the mean of the observations shown below?

Observations Frequencies
11 14
7 8
15 6
13 12

Explanation:

We have:

19992016 Manhattan Review www.manhattanreview.com


12 Sets & Statistics Guide Concepts

Observations Frequencies Observation Frequency


11 14 11 14 = 154
7 8 7 8 = 56
15 6 15 6 = 90
13 12 13 12 = 156
Total = 40 Total = 154 + 56 + 90 + 156 = 456

456
=> Mean = = 11.4
40

Alternate Approach:

Let the assumed mean be 11.

Thus, we have:

Modified Observation =
Observations Freq. Mod. Obs. Freq.
Observation Assumed mean
11 11 11 = 0 14 0 14 = 0
7 7 11 = 4 8 ( 4) 8 = 32
15 15 11 = 4 6 4 6 = 24
13 13 11 = 2 12 2 12 = 24
Total = 40 Total = 16
16
=> Mean = 11 + = 11.4
40

Median:

Median refers to the middle value of all observations arranged in ascending or descending
order.

Thus, we have:
n + 1 th
 
If n is odd: Median = Value of the observation.
2
n th n
   
If n is even: Median = Average of and + 1 th observations.
2 2

Let us take a few examples:

(1) What is the median of 123, 134, 128, 139, 141 and 126 ?

Explanation:

After arranging data in ascending order, we have:

www.manhattanreview.com 19992016 Manhattan Review


Sets & Statistics Guide Concepts 13

123, 126, 128, 134, 139, 141

Since there are 6 (even) terms, we have:

6 th 6
   
term + + 1 th term
2 2 3rd term + 4th term
Median = =
2 2

123, 126, 128 , 134 , 139, 141


3rd 4th

128 + 134
= = 131
2

(2) What is the median of the observations shown below?

Observations Frequencies
11 12
7 8
15 6
13 14

Explanation:

After arranging the observations in ascending order, we have:

Observations Frequencies Cumulative Frequency


7 8 8
11 12 12 + 8 = 20
13 14 14 + 20 = 34
15 6 6 + 34 = 40
Total = 40

Since there are 40 observations, we have:

40 40
   
th term + +1 th term
2 2 20th term + 21st term
Median = =
2 2

Observations Frequencies Cumulative Frequency


7 8 8
11 < 12 12 + 8 = 20 < 20th term
13 < 14 14 + 20 = 34 < 21st term
15 6 6 + 34 = 40
Total = 40

19992016 Manhattan Review www.manhattanreview.com


14 Sets & Statistics Guide Concepts

11 + 13
Median = = 12
2

Range:

Range refers to the difference between the values of maximum and minimum observations.

For example, the range of the set of observations 11, 13, 23, 21, 13, 17, 19 is 23 11 = 12

Standard Deviation:

Standard deviation shows the spread of the observations. If the observations are far away
from the mean, the standard deviation would be high; similarly, if the observations are close
to the mean, the standard deviation would be low.

For a set of n numbers x1 , x2 , x3 , . . . xn , the Standard Deviation (s) is calculated as:

If x is the mean of the observations:


s
(x1 x)2 + (x2 x)2 + + (xn x)2
s=
n

Let us take an example:

(1) What is the standard deviation of 12, 15, 16 and 17 ?

12 + 15 + 16 + 17
Mean = x = = 15
4
s
(12 15)2 + (15 15)2 + (16 15)2 + (17 15)2
=> s =
4
s
9+0+1+4
= = 3.5 = 1.87
4

A few important rules:

For a set of n numbers: x1 , x2 , x3 , . . . xn

If a constant number k is added to (or subtracted from) each term of the above set of
observations, we have:

The Mean increases (or decreases) by k


The Median increases (or decreases) by k
The Range remains unchanged

www.manhattanreview.com 19992016 Manhattan Review


Sets & Statistics Guide Concepts 15

The Standard Deviation remains unchanged

If a constant number k is multiplied (divided) to each term of the above set of observa-
tions, we have:

The Mean is multiplied (divided) by k


The Median is multiplied (divided) by k
The Range is multiplied (divided) by k
The Standard Deviation is multiplied (divided) by k

If a new term, whose value is equal to the mean, is introduced in the above set of obser-
vations, we have:

The Mean remains unchanged


Nothing can be concluded about Median; it depends on the distribution
The Range remains unchanged
The Standard Deviation decreases

If the terms of a set of observations arranged in ascending or descending order have a


constant difference between consecutive terms, i.e., they form an Arithmetic Sequence,
we have: Mean = Median

19992016 Manhattan Review www.manhattanreview.com


16 Sets & Statistics Guide Concepts

2.3 Data Interpretation

Data Interpretation mainly involves questions based on tables, charts, line graphs, bar graphs,
pie charts, etc. The data has to be assimilated and used to answer the questions that follow.
A single question can have multiple data forms. Thus, one may need to read the data quickly,
compare them and draw conclusions.

This is not simply a test of speedy calculation. One has to be able to derive logical inferences
from the data simultaneously, doing quick calculations.

In order to do calculations, one should be aware of calculation short-cut methods. The most
important calculation involves division and calculating the percentage, which takes up a lot of
time if done using conventional methods. Thus, the knowledge of short-cut division methods
and ratio comparison methods are very handy.

Fraction calculations

Comparing fractions: Different methods exist for comparing fractions. The methods are given
below:

Of two fractions, if their denominators are equal, the fraction with the greatest numera-
tor is the greatest.

Of two fractions, if their numerators are equal, the fraction with the smallest denomina-
tor is the greatest.

Of two fractions, if one fraction has greater numerator and smaller denominator, it is the
greater one.

5 9
For example: Between and , the latter is the greater fraction.
13 11

For fractions less than 1, if the difference between the numerator and its denominator is
equal for all the fractions, the fraction with the greatest numerator is the greatest.

2 5 9 11 11
For example: Among , , , and , the last fraction is the greatest.
3 6 10 12 12

For fractions greater than 1, if the difference between the numerator and its denominator
is equal for all the fractions, the fraction with the least numerator is the greatest.

3 6 10 12 3
For example: Among , , , and , the first fraction is the greatest.
2 5 9 11 2

www.manhattanreview.com 19992016 Manhattan Review


Sets & Statistics Guide Concepts 17

We can compare fractions by making the numerator equal to 1 for all fractions and cal-
culating approximate value of the denominator of each fraction.

5 9
For example: For the fractions and , we make the numerator 1 for both.
13 20
5 5/5 1 9 9/9 1
This gives us: = = and = =
13 13/5 2.6 20 20/9 2.2
9
 
Since the latter has a smaller denominator, it is a greater fraction.
20

Yet another method of comparing fractions is to cross-multiply the numerator of one


fraction and the denominator of the other fraction and then compare the products.

13 7
For example: For the fractions and
23 13

We multiply 13 and 13 to get 169 and on the other hand multiply 7 and 23 to get 161.

13 7
Since 169 > 161, we have: 13 13 > 7 23 => > .
23 13

19992016 Manhattan Review www.manhattanreview.com


18 Sets & Statistics Guide Concepts

Different ways of representing data:


Let us discuss the data given in a tabular form:

The following table gives the production of different electronic goods (in thousands) by a com-
pany X in the years 2014 and 2015:

A table:

Item 2013 2014


TVs 120 180
Laptops 240 270
Hard Disks 100 130
DVDs 140 120
Mobiles 200 300

The same data can be represented in different forms as shown below:

A pie chart:

Total production: 800 Total production: 1000

TVs TVs
15.00% 18.00%
Mobiles Mobiles
25.00% 30.00%

Laptops Laptops
DVDs 30.00%
17.50% DVDs 27.00%
12.00%
H. Disks H. Disks
12.50% 13.00%

2013 2014

The same data can be represented in the form of a bar graph as shown below:

A bar graph:

350

300

250
Production

200
2013
150 2014

100

50

0
TVs Laptops H. Disks DVDs Mobiles

www.manhattanreview.com 19992016 Manhattan Review


Sets & Statistics Guide Concepts 19

A stacked bar graph:

600

500

400
Production

300 2014
2013
200

100

0
TVs Laptops H. Disks DVDs Mobiles

A horizontal bar graph:

Mobiles

DVDs

H. Disks
2014
2013
Laptops

TVs

0 50 100 150 200 250 300 350


Production

19992016 Manhattan Review www.manhattanreview.com


20 Sets & Statistics Guide Concepts

A line graph:

350

300

250

200
Production

2013
150 2014

100

50

0
TVs Laptops H. Disks DVDs Mobiles

A scatter diagram:

350

300

250
Production

200
2013
150 2014

100

50

0
TVs Laptops H. Disks DVDs Mobiles

www.manhattanreview.com 19992016 Manhattan Review


Sets & Statistics Guide Concepts 21

Possible questions based on above graphs:

Table:

What is the percentage change in the production of TVs from 2013 to 2014?

Explanation:

Percentage change

Final value Initial value 180 120


= 100 = 100
Initial value 120

60
= 100 = 50%
120

Table:

What is the average production value of all electronic items (in thousands) in 2013?

Explanation:

Here, we simply need to take the average of all the production values in the year 2013.

120 + 240 + 100 + 140 + 200 800


Average = = = 160.
5 5

Bar graph:

Which item has the lowest percentage increase in its production from 2013 to 2014?

Explanation:

Here we need to compare the percentage changes for each item and select the minimum
value.

For minimum % change, the initial base value should be high and the corresponding
change should be low.

19992016 Manhattan Review www.manhattanreview.com


22 Sets & Statistics Guide Concepts

350

300

250
Production

200
2013
150 2014

100

50

0
TVs Laptops H. Disks DVDs Mobiles

We see that Laptops have a high initial value and the change is also small, so Laptops
could be a possible answer. Among the others, DVDs have a decrease, so cannot be the
answer, while the others have a relatively large change in their values.

Thus, the answer is Laptops.

Listing down the actual % changes for each item, we have:

Item 2013 2014 % change


TVs 120 180 50.0
Laptops 240 270 12.5
Hard Disks 100 130 30.0
DVDs 140 120 14.3
Mobiles 200 300 50.0

Thus, it is clear that the answer we obtained by reasoning is correct. Hence, we have
arrived at the correct answer without doing a single calculation.

Bar graph:

What is the net difference in total production (in thousands) between 2013 and 2014?

Explanation:

We can simply add up the values of 2013 and 2014 and subtract them.

Alternately, we can find the difference between each food item in each year and add up
the differences (with sign):

www.manhattanreview.com 19992016 Manhattan Review


Sets & Statistics Guide Concepts 23

Item 2013 2014 Difference


TVs 120 180 +60
Laptops 240 270 +30
Hard Disks 100 130 +30
DVDs 140 120 20
Mobiles 200 300 +100
Net difference 200

Line graph:

What is the maximum percentage change in production of any item from 2013 to 2014 ?

Explanation:

For this, we first need to identify which item has the maximum % change.

Looking at the line graph, it is clear that the item may be mobiles or TVs, since each has
a large difference in its two-year values.

350

300

250

200
Production

2013
150 2014

100

50

0
TVs Laptops H. Disks DVDs Mobiles

Percent increase in TVs

180 120
= 100 = 50%
120

Percent increase in Mobiles

300 200
= 100 = 50%
200

Thus, the answer is 50%.

19992016 Manhattan Review www.manhattanreview.com


24 Sets & Statistics Guide Concepts

Pie chart:

What is the angle subtended at the center by Laptops in 2013?

Explanation:

We need to convert the % values given in the pie chart to degrees.

We know that 100% corresponds to 360 .

Total production: 800 Total production: 1000

TVs TVs
15.00% 18.00%
Mobiles Mobiles
25.00% 30.00%

Laptops Laptops
DVDs 30.00%
17.50% DVDs 27.00%
12.00%
H. Disks H. Disks
12.50% 13.00%

2013 2014

In 2013, Laptops are represented by 30% in the pie chart.

Thus, the corresponding degree measure

30
360 = 108
100

Pie chart:

What is the % change in production of TVs from 2013 to 2014?

Explanation:

We need to understand that in the first pie chart, share of TVs is 15% of 800, while in the
second pie chart, it is 18% of 1000.

15
Thus, number of TVs in 2013 = 800 = 120
100
18
The number of TVs in 2014 = 1000 = 180
100

Thus, required percent change

www.manhattanreview.com 19992016 Manhattan Review


Sets & Statistics Guide Concepts 25

180 120
= 100 = 50%
120

Pie chart:

Compared to 2013, in 2014, production of which items has decreased?

Explanation:

We see that the total production, taking all the items, has increased from 2013 to 2014.

Hence, the items whose % values have gone up from 2013 to 2014 would imply an in-
crease in production.

Only those items whose % values have decreased would possibly have decreased produc-
tion values.

Thus, possible items are Laptops (decreased from 30% to 27.5%) and DVDs (decreased
from 17.5% to 12%).

Laptops:

30
The number of Laptops in 2013 = 800 = 240
100
27
The number of Laptops in 2014 = 1000 = 270
100

Thus, the number of Laptops has rather increased.

DVDs:

17.5
The number of DVDs in 2013 = 800 = 140
100
12
The number of DVDs in 2014 = 1000 = 120
100

Thus, only DVDs production has decreased.

Pie chart:

If it was found that the production of TVs has been under-reported by 40% in 2013, what
is actual percentage share of TVs in 2013?

19992016 Manhattan Review www.manhattanreview.com


26 Sets & Statistics Guide Concepts

Explanation:

Assuming the total production as 100 units, production of TVs 15% of 100 = 15 units.

Since TVs have been under-reported by 40%, we can say that 15 units correspond to 100%
40% = 60% of the actual production of TVs.

Hence actual production of TVs

15
100 = 25
60

Thus, the number of TVs have increased in value by 25 15 = 10 units

Thus, the total would also increase by 10 units and would become 110 units

Thus, actual percentage share of TVs

25 250 8
= 100 = % = 22 %
110 11 11

Pie chart:

If it was found that the production of Laptops has been under-reported by 20% in 2013,
which of the following options can represent the actual % share of any items in 2013?

(A) 3.3%
(B) 23.3%
(C) 28.2%
(D) 41.5%

Explanation:

Since the production of Laptops has been under-reported, its actual % share is going to
increase and hence, the % share of the other items would decrease.

Assuming the total production equal to 100 units, production of Laptops = 30% of 100 =
30 units

Since Laptops have been under-reported by 20%, we can say that 30 units correspond to
80% of the actual production.

www.manhattanreview.com 19992016 Manhattan Review


Sets & Statistics Guide Concepts 27

30
Hence actual production of Laptops = 100 = 37.5 units
80

Thus, Laptops have increased in value by 37.5 30 = 7.5 units

Thus, the total would also increase by 7.5 units and would become 107.5 units.

Since the increase in the total is only slight, the % share of Laptops would increase and
that of the others would decrease only by a very slight amount.

25
 
Thus, the only possible % could be 23.3% = 100% , that of Mobiles, when it
107.5
reduces from 25%.

Pie chart:

If the % distribution of production of the items is the same in the year 2015 as it is in
2014, while the total production increases by 20% from that in 2014, what would be the
production value of Mobiles in 2015?

Explanation:

Total production in 2015 = 1000 + 20% of 1000 = 1200

Since the % distribution of the items remains the same, production of Mobiles

30
= 30% of 1200 = 1200 = 360
100

Scatter plot:

If the ratio of selling price of a Mobile to that of a Laptop is 1 : 2, what is the ratio of the
sales proceeds of Mobiles to those of Laptops in 2014?

Explanation:

19992016 Manhattan Review www.manhattanreview.com


28 Sets & Statistics Guide Concepts

350

300

250
Production

200
2013
150 2014

100

50

0
TVs Laptops H. Disks DVDs Mobiles

Production of Mobiles in 2014 = 300

Production of Laptops in 2014 = 270

Thus, ratio of production of Mobiles to that of Laptops

300 10
= =
270 9
1
Ratio of selling price of a Mobile to that of a Laptop =
2

Hence, ratio of sales proceeds of Mobiles to Laptops

10 1 5
= =
9 2 9

Scatter plot:

If the % change in production of Hard Disks from 2014 to 2015 is the same as that in the
2013 to 2014 and the production of Hard Disks represents 10% of the total production
in 2015, what is the total production (in thousands) of all items in 2015?

Explanation:

We know that

Hard Disks production in 2013 = 100

Hard Disks production in 2014 = 130

www.manhattanreview.com 19992016 Manhattan Review


Sets & Statistics Guide Concepts 29

Thus, % change in Hard Disks production from 2013 to 2014

130 100
100 = 30%
100

Since the % increase in 2015 remains the same, the production of Hard Disks in 2015

= 130 + 30% of 130 = 130 + 39 = 169

Since this represents 10% of total production in 2015, total production

169
= 100 = 1690
10

19992016 Manhattan Review www.manhattanreview.com


30 Sets & Statistics Guide Concepts

In the GMAT, only two kinds of questions asked: Problem Solving and Data Sufficiency.

Problem Solving
Problem solving (PS) questions may not be new to you. You must have seen these types of
questions in your school or college days. The format is as follows: There is a question stem
and is followed by options, out of which, only one option is correct or is the best option that
answers the question correctly.

PS questions measure your skill to solve numerical problems, interpret graphical data, and
assess information. These questions present to you five options and no option is phrased as
None of these. Mostly the numeric options, unlike algebraic expressions, are presented in an
ascending order from option A through E, occasionally in a descending order until there is a
specific purpose not to do so.

Data Sufficiency
For most of you, Data Sufficiency (DS) may be a new format. The DS format is very unique to the
GMAT exam. The format is as follows: There is a question stem followed by two statements,
labeled statement (1) and statement (2). These statements contain additional information.

Your task is to use the additional information from each statement alone to answer the ques-
tion. If none of the statements alone helps you answer the question, you must use the infor-
mation from both the statements together. There may be questions which cannot be answered
even after combining the additional information given in both the statements. Based on this,
the question always follows standard five options which are always in a fixed order.

(A) Statement (1) ALONE is sufficient, but statement (2) ALONE is not sufficient to answer the
question asked.

(B) Statement (2) ALONE is sufficient, but statement (1) ALONE is not sufficient to answer the
question asked.

(C) BOTH statements (1) and (2) TOGETHER are sufficient to answer the question asked, but
NEITHER statement ALONE is sufficient to answer the question asked.

(D) EACH statement ALONE is sufficient to answer the question asked.

(E) Statements (1) and (2) TOGETHER are NOT sufficient to answer the question asked, and
additional data specific to the problem are needed.

In the next chapters, you will find 100 exam-like quants questions. Best of luck!

www.manhattanreview.com 19992016 Manhattan Review


Chapter 3

Questions

31
32 Sets & Statistics Guide Questions

3.1 Problem Solving Questions

3.1.1 Sets
1. Each person who attended a company meeting either took only one glass of juice or only
one glass of wine, or one glass of each. Of those who attended the meeting, 62 percent
took one glass of wine and 53 percent did not take juice. If the total number of persons
who attended the meeting be 100, how many glasses were used to serve the drinks if
each glass is to be used only once?

(A) 56
(B) 71
(C) 91
(D) 100
(E) 109

Solve yourself:

2. Last year, 36 houses in a certain development had roof repairs and 48 houses were
repainted. If 20 houses in the development had roof repairs but were not repainted last
year, how many houses were repainted but did not have roof repairs?

(A) 12
(B) 16
(C) 20
(D) 28
(E) 32

Solve yourself:

3. There are a total of 66 students in a school, who have to study at least one of three
subjects Mathematics, Physics and Economics. There are 28 students, who did not take
up either Physics or Economics; 15 students, who did not take up either Mathematics or
Economics; 45 students, who did not take up Economics; and 30 students, who did not
take up Mathematics. The number of students, who did not take up either Mathematics

www.manhattanreview.com 19992016 Manhattan Review


Sets & Statistics Guide Questions 33

or Economics, exceeded the number of students, who did not take up Mathematics or
Physics by 2. If 5 students took up all three subjects, how many students took up both
Economics and Mathematics?
(A) 1
(B) 5
(C) 6
(D) 7
(E) 10
Solve yourself:

4. In a survey of 63 people, 33 people subscribed to magazine A, 30 people subscribed to


magazine B and 17 subscribed to magazine C. For any two of the magazines, 9 people
subscribed to both magazines. If 5 people in the survey did not subscribe to any of the
three magazines, how many people subscribed to all three magazines?
(A) 10
(B) 9
(C) 7
(D) 5
(E) 2
Solve yourself:

5.

Factor Percent of respon-


dents

User-friendly 56%

Fast response time 48%

Bargain prices 42%

19992016 Manhattan Review www.manhattanreview.com


34 Sets & Statistics Guide Questions

The table above gives three factors to be considered when choosing an Internet service
provider and the percent distribution of the 1,200 respondents who cited those factors
as important, according to a survey. If 30 percent of the respondents cited both User
friendly" and Fast response time" as important factors, what is the maximum possible
number of respondents who cited Bargain prices", but neither User-friendly" nor Fast
response time" as an important factor?
(A) 312
(B) 336
(C) 360
(D) 384
(E) 420
Solve yourself:

6. A survey shows that 63% of the people in New York like pasta, whereas 76% like burgers.
If x% of the people like pasta and burgers both, which of the following statements is
correct?
I. 39 x 63
II. Maximum percent of people who like only one between pasta and burgers is 61%
III. Maximum percent of people who do not like either pasta or burgers is 24%

(A) Only I
(B) Only II
(C) Both I and II
(D) Both II and III
(E) I, II and III
Solve yourself:

7. In a trip to a theme park, 30 children take rides among Merry-go-round, Roller-coaster


and Toy-train. A child can take multiple rides but at most one ride of any type. There
are 11 children who ride the Merry-go-round and 14 children ride the Roller-coaster.

www.manhattanreview.com 19992016 Manhattan Review


Sets & Statistics Guide Questions 35

There are 10 children who take two different rides, 2 children take all three rides and all
children take at least one ride. If each ride costs $5, what is the total cost of all the rides
taken by the 30 children?

(A) $44
(B) $124
(C) $150
(D) $210
(E) $220

Solve yourself:

8. In a class, 36% of the students failed in the English test and 30% in the Mathematics test.
What is the sum of the maximum and the minimum percentages of students who passed
in both English and Mathematics?

(A) 34
(B) 54
(C) 66
(D) 98
(E) 120

Solve yourself:

9. A, B and C are three TV channels. A survey shows that 30%, 20% and 85% of the people in
a locality watch the channels A, B and C, respectively. 20% of people watch exactly two of
the three channels and 5% of the people watch none of the three channels. What percent
of the people watch all the three channels?

(A) 0
(B) 5
(C) 10
(D) 15
(E) 20

19992016 Manhattan Review www.manhattanreview.com


36 Sets & Statistics Guide Questions

Solve yourself:

www.manhattanreview.com 19992016 Manhattan Review


Sets & Statistics Guide Questions 37

3.1.2 Statistics

Questions on Average (Arithmetic Mean)

10. If all the elements of a set are multiplied by a constant, the average (arithmetic mean)
remains unchanged, which of the following statements must true?
I. Average (arithmetic mean) = 0
II. Sum of the elements of the set equals 0.
III. The set contains at least one positive and one negative number

(A) I only
(B) II only
(C) III only
(D) I, and II only
(E) I, II, and III
Solve yourself:

11. Average (arithmetic mean) of x and y is 80, and the average of x and z is 200; what is
(z y)
the value of ?
8

(A) 80
(B) 30
(C) 0
(D) 30
(E) 120
Solve yourself:

12. Three numbers have their average (arithmetic mean) equals to 6. The middle number
is 7. If the average of two larger numbers is 8, what is the average of the two smaller
numbers?

19992016 Manhattan Review www.manhattanreview.com


38 Sets & Statistics Guide Questions

(A) 2
(B) 4
(C) 4.5
(D) 6
(E) 6.5
Solve yourself:

13. The average (arithmetic mean) of six numbers is 7.5. If one of the six numbers is mul-
tiplied by 4, the average of the numbers decreases by 0.5, which of the six numbers is
multiplied by 4?
(A) 3
(B) 2
(C) 0
(D) 2
(E) 3
Solve yourself:

14. A students average (arithmetic mean) score on 5 tests is 38. What must be the students
score on the 6th test for the average score on all the 6 tests to be 41?
(A) 40
(B) 44
(C) 50
(D) 54
(E) 56
Solve yourself:

www.manhattanreview.com 19992016 Manhattan Review


Sets & Statistics Guide Questions 39

15. For the first x days, the average (arithmetic mean) rainfall was 32 millimeter per day. If
todays rainfall of 56 millimeter increased the average rainfall to 36 millimeter per day,
what is the value of x?
(A) 2
(B) 3
(C) 5
(D) 6
(E) 7
Solve yourself:

16. A student appeared for a number of tests, each test having maximum marks of 200. His
average score is 70 greater than his lowest score in any test. However, the median score
is 50 greater than the lowest score in any test. If the range of scores of the student is
160, and the number of tests is an odd number, what is the minimum number of tests
that the student appeared for?
(A) 3
(B) 5
(C) 7
(D) 9
(E) 11
Solve yourself:

17. The average of 7 numbers is 12. The average of the 4 smallest numbers in this set is 8,
while the average of the 4 greatest numbers in this set is 20. How much greater is the
sum of the 3 greatest numbers than the sum of the 3 smallest numbers?
(A) 32
(B) 46
(C) 48
(D) 80
(E) 84

19992016 Manhattan Review www.manhattanreview.com


40 Sets & Statistics Guide Questions

Solve yourself:

18. The average age of the members of a club is 32 years. If four of the members are 25
years old and none of the members are more than 60 years old, which of the following
could be the number of members in the club?

I. 5
II. 6
III. 9

(A) Only I
(B) Only II
(C) Only III
(D) Both II and III
(E) I, II and III

Solve yourself:

19. The average of five distinct integers is 65. If the largest integer is 75, what is the maxi-
mum possible value of the smallest integer?

(A) 60
(B) 61
(C) 62
(D) 63
(E) 64

Solve yourself:

www.manhattanreview.com 19992016 Manhattan Review


Sets & Statistics Guide Questions 41

20. Four students contributed to a charity drive, and the average amount contributed by
the students was $20. If no student gave more than $25 and all contributed different
amounts in a whole of a dollar, what is the maximum ratio of the amounts contributed
by any two students?

(A) 3
1
(B) 3
8
5
(C) 3
8
(D) 5
1
(E) 5
2
Solve yourself:

21. The mean of n terms is x. If the first term is increased by 1, the second by 2, the third
by 3, and so on, which of the following denotes the new mean?
n1
(A) x+
2
n
(B) x+
2
n+1
(C) x+
2
(D) x+n1
(E) x+n

Solve yourself:

19992016 Manhattan Review www.manhattanreview.com


42 Sets & Statistics Guide Questions

Questions on Median

22. The number of defects in the first five cars to come through a new production line are 9,
7, 10, 4, and 6, respectively. If the sixth car through the production line has n defects,
for which of these values of n, the mean number of defects per car for the first six cars
equals their median?
I. 3
II. 7
III. 12

(A) Only I
(B) Only II
(C) Only III
(D) Only I and III
(E) I, II and III
Solve yourself:

23. Set X consists of the following unique integers: {1, 18, 4, p, 3, 16, 2, 26}; which of the
following CANNOT be the median of set X?
(A) 1
(B) 1.5
(C) 2
(D) 2.5
(E) 3
Solve yourself:

24. Which of the following number CANNOT be included in the set of the following numbers
to decrease the median of the set by 1.5?

Set: {28, 18, 22, 21, 7, 13}

www.manhattanreview.com 19992016 Manhattan Review


Sets & Statistics Guide Questions 43

(A) 7
(B) 10
(C) 11.5
(D) 18
(E) 18.5

Solve yourself:

25. If two sets, set X: {2, 23, 4, 8, 6}, and set Y: {0, 2, 8, 8, 12, 13, y} have equal median
values, what is the value of y?

(A) 0
(B) 2
(C) 3
(D) 4
(E) 8

Solve yourself:

26. If the median of X consecutive integers is 50.5, which is the smallest integer among the
following?

(A) 51 X/2
(B) 50.5 X/2
(C) 50 X/2
(D) 49.5 X/2
(E) 49 X/2

Solve yourself:

19992016 Manhattan Review www.manhattanreview.com


44 Sets & Statistics Guide Questions

27. Five people have few marbles with them. The numbers of marbles with them are 23, 45,
51, 66 and 73. If they redistribute the marbles among themselves such that the number
of marbles with each is a distinct integer and the median value of the number of marbles
is maximized, what is the range of the number of marbles with them?

(A) 83
(B) 84
(C) 85
(D) 86
(E) 88

Solve yourself:

28. The heights of students in a class are measured. It is found that, for boys, the average
height is (3x + 1) feet; while, for the girls, the average height is (6 2x) feet. If there are
a total of 5x students, with boys and girls in the ratio 2 : 3. What is the average height
of all students taken together?

(A) 4 feet
(B) 4x feet
(C) 20 feet
(D) 20x feet
(E) 12x 2 feet

Solve yourself:

29. The heights of students in a class are measured. It is found that, for boys, the average
height is (3x + 1) feet; while, for the girls, the average height is (6 2x) feet. If there are
a total of 5x students, with boys and girls in the ratio 2 : 3. What is the average height
of all students taken together?

(A) 4 feet
(B) 4x feet
(C) 20 feet
(D) 20x feet

www.manhattanreview.com 19992016 Manhattan Review


Sets & Statistics Guide Questions 45

(E) 12x 2 feet

Solve yourself:

30. A survey is conducted on monthly salaries of households of a locality. For each house-
hold, the salary of the eldest member is considered. The data is then grouped according
to different age-groups. From each age-group, the median salary is considered as shown
in the table below.

Between 15 Between 25 Between 35 Between 145 Between 55


Age-group
and 25 years and 35 years and 45 years and 55 years and 65 years

Median Salary $12000 $24000 $36000 $38000 $28000

If the data for the age groups 15-25 years and 55-65 years are ignored, which of the
following statements would be correct?

I. The mean of the above median salaries would change by less than $5000
II. The median of the above median salaries would change by less than $5000
III. The difference between the new mean and new median of the above median salaries
is less than $5000

(A) Only I
(B) Only II
(C) Only III
(D) Both I and II
(E) I, II and III

Solve yourself:

31. If x and y are positive integers and the median of 3x1 5y , 3x 5y1 and 3x 5y is 2025,
what is the value of xy?

(A) 8

19992016 Manhattan Review www.manhattanreview.com


46 Sets & Statistics Guide Questions

(B) 9
(C) 10
(D) 12
(E) 15

Solve yourself:

www.manhattanreview.com 19992016 Manhattan Review


Sets & Statistics Guide Questions 47

Questions on Average and Median

32. The average (arithmetic mean) of five positive integers k, m, r , s and t is 16, and k <
m < r < s < t. If t is 40, what is the greatest possible value of the median of the five
integers?
(A) 16
(B) 18
(C) 19
(D) 20
(E) 38
Solve yourself:

33. On Monday, 9 students each took a test having 100 questions. The average (arithmetic
mean) number of the correct answers was 50, and the median number of correct answers
was 40. Which of the following statements must be true?
I. At least one student had more than 60 correct answers.
II. At least one student had more than 40 and less than 50 correct answers.
III. At least one student had less than 40 correct answers.

(A) Only I
(B) Only II
(C) Only III
(D) Only I and III
(E) Only II and III
Solve yourself:

34. Nine boys have an average (arithmetic mean) weight of 36 kg and a median weight of 38
kg. If the weight of the heaviest boy is 3 kg less than 3 times the weight of the lightest
boy, what is the maximum possible weight, in kg, of the heaviest boy?
(A) 38

19992016 Manhattan Review www.manhattanreview.com


48 Sets & Statistics Guide Questions

(B) 54
(C) 72
(D) 94
(E) 100
Solve yourself:

35. Number of defects in eight of the nine batches of batteries are: 2, 4, 6, 5, 3, 3, 7, 8,


respectively. If the median defect per batch for all the nine batches equals the average
(arithmetic mean) defect per batch for all the nine batches, which of the following would
be the number of defects in the ninth batch?
(A) 2
(B) 3
(C) 4
(D) 5
(E) 7
Solve yourself:

  
36. For the set of numbers
 x, y, x + y , x 4y , xy, 2y , if y > 6 and the mean of the
set equals y + 3 , which of the following denotes the median of the above set?
(A) x+y
x
(B) +y
2
y
(C) x +
2
(D) x 4y
x+y
(E)
2
Solve yourself:

www.manhattanreview.com 19992016 Manhattan Review


Sets & Statistics Guide Questions 49

37. Four positive integers, a, b, c and d are such that a < b < c < d. The mean of the above
numbers is 12 and the median is 7. If one of the numbers is 8, what is the minimum
possible value of d?

(A) 26
(B) 27
(C) 28
(D) 29
(E) 30

Solve yourself:

38. If the mean of 3x1 , 3x1 5 and 3x 52 is 243, what is the median of the above numbers?

(A) 15
(B) 45
(C) 81
(D) 135
(E) 225

Solve yourself:

19992016 Manhattan Review www.manhattanreview.com


50 Sets & Statistics Guide Questions

Questions on Range

39. Which of the following could be the range of set { 43, 76, a, b, c}?
I. 129
II. 112
III. 154

(A) I only
(B) II only
(C) III only
(D) I or II only
(E) II or III only
Solve yourself:

40. For a set of 8 positive distinct integers, the mean, median, and range are calculated. If
the smallest three numbers of the set are reduced by 2 each and the largest two are
increased by 3 each, which of the following statements is correct?
I. The mean of the set of numbers would change
II. The median of the set of numbers would change
III. The range of the set of numbers would change

(A) Only I
(B) Only II
(C) Only III
(D) Both I and II
(E) Both I and III
Solve yourself:

41. The range of heights of students in Class A is 9 inches and that for Class B is 6 inches.
The information contained in which of the following statements in itself may be sufficient
to determine the range of height of all the students from classes A and B?

www.manhattanreview.com 19992016 Manhattan Review


Sets & Statistics Guide Questions 51

I. The tallest student in Class B is 5 inches taller than the tallest student in Class A
II. The shortest student in Class B is 1 inches shorter than the tallest student in Class
A
III. The shortest student in Class A is 8 inches shorter than the shortest student in
Class B

(A) Only I
(B) Only II
(C) Only III
(D) Both I and II
(E) I, II and III

Solve yourself:

19992016 Manhattan Review www.manhattanreview.com


52 Sets & Statistics Guide Questions

Questions on Standard deviation

42. John, a student of the ninth grade, scored 75 in an exam where the mean score of all
students was 70 and standard deviation of the scores was 2. Bob, a student of the tenth
grade, scored 72 in an exam where the mean score of all students was 65 and standard
deviation of the scores was 3. Which of the following statements is correct?
I. Johns performance in his exam was better than Bobs performance in his exam.
II. There was at least one student who scored more than John.
III. There was at least one student who scored less than Bob.

(A) Only I
(B) Only II
(C) Only III
(D) Only I and III
(E) Only II and III
Solve yourself:

43. Which of the following set(s) have a standard deviation less than the standard deviation
of set X: {10, 20, 30, 40}?

P = {2,12, 22, 32}


Q = {3, 7,17, 27}
R = {6, 4, 14, 24}

(A) Set P
(B) Set Q
(C) Set R
(D) Set P, and R
(E) None
Solve yourself:

www.manhattanreview.com 19992016 Manhattan Review


Sets & Statistics Guide Questions 53

44. If set X is {p, q, r }, which of the following sets must have the same standard deviation
as has set X?

(A) {p + 1, q, r }
(B) {p, q + 1, r }
(C) {p, q, r + 1}
(D) {p + qr , q(1 + r ), r (q + 1)}
(E) {p 1, q + 1, r }

Solve yourself:

45. Sets A, B and C are given below. If a number "100" is added to each element of each
set, which of the following represents the correct ordering of the sets in terms of the
absolute increase in their Standard Deviation, from the largest to the smallest?

A = {42, 32, 22,12, 2}


B = {95, 90, 85, 80, 75}
C = {40, 20, 0, 20, 40}

(A) ABC
(B) BCA
(C) CAB
(D) ACB
(E) CBA

Solve yourself:

46. If one of P and Q is a positive number and the other is a negative number, which of the
following could be the standard deviation of set {P , 12, 8, 4, 0, 4, 8, Q}?

I. 2
II. 4
III. 0

19992016 Manhattan Review www.manhattanreview.com


54 Sets & Statistics Guide Questions

(A) I only
(B) II only
(C) I, and II only
(D) I, and III only
(E) None of these

Solve yourself:

47. Set A consists of 25 numbers with average (arithmetic mean) 5 and standard deviation
4. If two elements {3, 5} are swapped with the following elements, which of them would
most decrease the standard deviation?

(A) {0, 8}
(B) {5, 3}
(C) {2, 6}
(D) {4, 4}
(E) {1, 7}

Solve yourself:

48. Distribution of test scores of a large number of test takers is symmetric about the mean
score . If 95 percent of the distribution lies within 2 standard deviation of the mean
score, what percent of the distribution is more than 2 ?

(A) 2.5%
(B) 5%
(C) 84%
(D) 95%
(E) 97.5%

www.manhattanreview.com 19992016 Manhattan Review


Sets & Statistics Guide Questions 55

Solve yourself:

19992016 Manhattan Review www.manhattanreview.com


56 Sets & Statistics Guide Questions

3.1.3 Data Interpretation

49. According to the chart given below, which of the following is closest to the median
number of unemployed for the years 2001 to 2011, inclusive?

Number of unemployed
22500
20000
17500
15000
12500
10000
7500
5000
2500
0
01

02

03

04

05

06

07

08

09

10

11
20

20

20

20

20

20

20

20

20

20

20
Year

(A) 11500

(B) 12500

(C) 17000

(D) 18000

(E) 20000

Solve yourself:

50.

www.manhattanreview.com 19992016 Manhattan Review


Sets & Statistics Guide Questions 57

Country Consumption (million kilograms)

J 1080

K 600

L 360

M 330

N 310

Total 2680

The table above gives the coffee consumption in 1994 for five countries. If the total coffee
consumption of these countries was 40 percent of the worlds coffee consumption, the
coffee consumption of country M was what percent of the coffee consumption of the
countries not shown in the above table?
(A) 8.3%
(B) 12.5%
(C) 15.0%
(D) 20.0%
(E) 22.5%
Solve yourself:

51.

Brand Brand
X Y

Miles per gal- 40 36


lon

Cost per gal- $2.40 $2.25


lon

19992016 Manhattan Review www.manhattanreview.com


58 Sets & Statistics Guide Questions

The table above gives the gasoline costs and consumption rates for a certain car driven
at 50 miles per hour, using each of two brands of gasoline. How many miles farther can
the car be driven at this speed on $36 worth of Brand X gasoline than on $36 worth of
Brand Y gasoline?

(A) 20
(B) 24
(C) 84
(D) 100
(E) 104

Solve yourself:

52.

Sector Net Income (billion Percent change


$), from
second quarter, 2006 first quarter, 2006

Basic materi- 48 20%


als

Energy 70 +40%

Industrial 45 10%

Utilities 120 +300%

Conglomerates 24 +20%

The table above represents the combined net income of all United States companies
operating in each of five sectors for the second quarter of 2006. Which sector had the
greatest net income during the first quarter of 2006?

(A) Basic materials


(B) Energy
(C) Industrial
(D) Utilities
(E) Conglomerates

www.manhattanreview.com 19992016 Manhattan Review


Sets & Statistics Guide Questions 59

Solve yourself:

53.

Month Number
of days
worked

June 20

July 17

August 19

The table above shows the number of days worked by a certain sales representative in
each of three months last year. If the number of sales calls that the representative made
each month was proportional to the number of days worked in that month, and if the
representative made a total of 168 sales calls in the three months shown, how many sales
calls did the representative make in August?

(A) 50

(B) 51

(C) 56

(D) 57

(E) 60

Solve yourself:

54.

19992016 Manhattan Review www.manhattanreview.com


60 Sets & Statistics Guide Questions

Age cat- Number of


egory (in employees
years)

Less than 20 29

20 29 58

30 39 36

40 49 21

50 59 10

60 69 5

70 and 2
above

There are 161 employees at Company X. The above table shows the age category of all
the employees. According to the table, if m is the median age, in years, of the employees
at Company X, then m must satisfy which of the following?

(A) 20 m 29

(B) 25 m 34

(C) 30 m 39

(D) 35 m 44

(E) 40 m 49

Solve yourself:

55.

www.manhattanreview.com 19992016 Manhattan Review


Sets & Statistics Guide Questions 61

12

Revenue (in billions of dollars)


10

6
11.3
10
4 8
6
2

0
1993 1994 1995 1996

The graph above shows the combined revenue, in billions of dollars, of a chain of food
stores for each year over a four-year period. In 1994 a certain stores revenue accounted
for 2.0 percent of the combined revenue for that year, and in 1995 the same store ac-
counted for 2.3 percent of the combined revenue for that year. What was the approximate
percent increase in revenue for this store from 1994 to 1995?
(A) 0.3%
(B) 15.0%
(C) 25.0%
(D) 30.4%
(E) 43.8%
Solve yourself:

56. The ratio of imports to exports of a particular company X is shown in the graph above. If
the exports were $60 million in 2001 and the exports increased by $2 million every year,
what was the percent increase in imports in 2006 over 2001?
1.6

1.4

1.2

1.0

0.8

0.6

0.4

0.2
2001 2002 2003 2004 2005 2006

19992016 Manhattan Review www.manhattanreview.com


62 Sets & Statistics Guide Questions

(A) 45.5%
(B) 55.5%
(C) 60.0%
(D) 75.0%
(E) 88.8%
Solve yourself:

57.

2011 2012

Attribute Bank X Bank Y

Loan Sanc- 650 1500


tions

Total Revenue 1600 3500

The table above shows the performance of two banks, Bank X and Bank Y (all values
are in million $) for the year 2011 2012. What is the positive difference between Loan
Sanctions as a percent of Total Revenue for Bank X and Bank Y?
(A) 1.3
(B) 2.2
(C) 3.0
(D) 3.6
(E) 4.2
Solve yourself:

58.

www.manhattanreview.com 19992016 Manhattan Review


Sets & Statistics Guide Questions 63

Price of balloons

Packet size Price per


packet

1 balloon $1

10 balloons $9

100 balloons $75

1000 balloons $600

Balloons are priced according to the chart above. If Charles wants to buy 2000 balloons,
how much money does he save by buying 1000 balloons at a time rather than ten balloons
at a time?

(A) $100
(B) $200
(C) $300
(D) $600
(E) $1200

Solve yourself:

59.

500
450
400
350
300
250
200
150
100
50
0
Indiana Vegas Michigan Florida

19992016 Manhattan Review www.manhattanreview.com


64 Sets & Statistics Guide Questions

The bar-chart above shows the number of high speed internet customers (all values are in
thousands) for a particular company in four states in 2011. If the number of customers
in Indiana and Michigan together was 20 percent of the total number of customers of the
company in 2011, the number of customers in Florida represents what percent of the
total customers of the company in 2011?

(A) 5.0%
(B) 10.3%
(C) 16.4%
(D) 22.5%
(E) 30.0%

Solve yourself:

60.

Finances of company X

Attribute 2010 2011

Total Income 250 450

Net Profit 125 150

Cash Profit 80 100

The above table shows the finances of Company X (all values are in million $) for the
years 2010 and 2011. If the percent increase in Net Profits from 2010 to 2011 was the
same as that from 2011 to 2012, while the total income of the company doubled from
2011 to 2012, what was the ratio of Net Profits to Total Income of the company in 2012?
1
(A)
5
1
(B)
4
1
(C)
3
1
(D)
2

www.manhattanreview.com 19992016 Manhattan Review


Sets & Statistics Guide Questions 65

4
(E)
5
Solve yourself:

19992016 Manhattan Review www.manhattanreview.com


66 Sets & Statistics Guide Questions

3.2 Data Sufficiency Questions

Data sufficiency questions have five standard options. They are listed below and will not
be repeated for each question.

(A) Statement (1) ALONE is sufficient, but statement (2) ALONE is not sufficient to an-
swer the question asked.
(B) Statement (2) ALONE is sufficient, but statement (1) ALONE is not sufficient to an-
swer the question asked.
(C) both the statements (1) and (2) TOGETHER are sufficient to answer the question
asked, but NEITHER statement ALONE is sufficient to answer the question asked.
(D) EACH statement ALONE is sufficient to answer the question asked.
(E) Statements (1) and (2) TOGETHER are NOT sufficient to answer the question asked,
and additional data specific to the problem are needed.

www.manhattanreview.com 19992016 Manhattan Review


Sets & Statistics Guide Questions 67

3.2.1 Sets

61. S is a set of integers. Is there a pair of integers belonging to set S, having a sum of 102?

(1) The set S has 27 distinct positive odd numbers.


(2) The maximum number in set S is less than 100.

Solve yourself:

62. In a class of 50 students, 20% of the boys cleared a test. How many total students cleared
the test?

(1) 40% of the students are girls.


(2) The number of boys who did not clear the test is twice the number of girls who
cleared the test.

Solve yourself:

19992016 Manhattan Review www.manhattanreview.com


68 Sets & Statistics Guide Questions

3.2.2 Statistics

Questions on Average (Arithmetic Mean)

63. What is the median of three numbers whose average (arithmetic mean) is 120?

(1) Average (arithmetic mean) of the first two numbers is 110


(2) Average (arithmetic mean) of the last two numbers is 110

Solve yourself:

64. On the number line below, a, b, c, d, e, f and g are seven consecutive odd integers in
increasing order. What is the average (arithmetic mean) of these seven integers?

a b c d e f g

(1) a + e = 34
(2) c + g = 42

Solve yourself:

65. If the smallest of the seven numbers is 4, is the average (arithmetic mean) of seven
numbers less than 12?

(1) The average (arithmetic mean) of four largest numbers is 18.25.


(2) The average (arithmetic mean) of three smallest numbers is 8.33.

Solve yourself:

www.manhattanreview.com 19992016 Manhattan Review


Sets & Statistics Guide Questions 69

66. If the average (arithmetic mean) of five numbers is 100, how many of them are equal to
100?

(1) None of the five numbers is less than 99.


(2) None of the five numbers is greater than 101.

Solve yourself:

67. This is a modified version of the previous question.

If the average (arithmetic mean) of five numbers is 100, how many of them are equal to
100?

(1) All the five numbers are greater than 99.


(2) All the five numbers are less than 101.

Solve yourself:

68. What if the above question says "... the five numbers are integers?"
Solve yourself:

19992016 Manhattan Review www.manhattanreview.com


70 Sets & Statistics Guide Questions

Questions on Median

69. Is the median of set X greater than its average (arithmetic mean)?
(1) Set X consists of consecutive multiples of 3.
(2) Sum of all the elements of set X is 84.
Solve yourself:

70. Set X is formed out of including all the terms of sets P, Q, and R; it has no other terms
except the terms from sets P, Q, and R. What is the median of the set X?
(1) Each of sets P, Q, and R have a median of 42.
(2) Each of sets P, Q, and R have equal number of terms.
Solve yourself:

71. What is the value of q in the list given below?

p, q, 8, 10, 12, 14

(1) pq
(2) Median value of the list is 10
Solve yourself:

72. What is the median number of players participating per team in the Grand Soccer League
event?
(1) 60 percent of the teams in the Grand Soccer League event have 22 or more players
participating in each team.

www.manhattanreview.com 19992016 Manhattan Review


Sets & Statistics Guide Questions 71

(2) 60 percent of the teams in the Grand Soccer League event have 22 or fewer players
participating in each team.

Solve yourself:

73. What if the question were:

What is the median number of players participating per team in the Grand Soccer League
event?

(1) 60 percent of the teams in the Grand Soccer League event have 22 or more players
participating in each team.
(2) 40
 60 percent of the teams in the Grand Soccer League event have 22 or fewer
players participating in each team.

Solve yourself:

74. What if the question were:

What is the median number of players participating per team in the Grand Soccer League
event?

(1) 60
 40 percent of the teams in the Grand Soccer League event have 
22
 23 or more
players participating in each team.
(2) 60
 40 percent of the teams in the Grand Soccer League event have 21 or fewer
players participating in each team.

Solve yourself:

19992016 Manhattan Review www.manhattanreview.com


72 Sets & Statistics Guide Questions

Questions on Average and Median

75. If average (arithmetic mean) score of a batch of four students is 80, how many students
have scored less than 80?

(1) Two top scorers have scored 120 each


(2) Median score of the batch is 80

Solve yourself:

76. What is the median of four positive integers whose average (arithmetic mean) is 90?

(1) Average (arithmetic mean) of the first, second and fourth integer is 90.
(2) Average (arithmetic mean) of the first, third and fourth integer is 90.

Solve yourself:

77. This is a modified version of the previous question.

What is the median of four positive integers whose average (arithmetic mean) is 90?

(1) Average (arithmetic mean) of the first, second and fourth integers is 110.
(2) Average (arithmetic mean) of the first, third and fourth integers is 80.

Solve yourself:

78. In the set of positive integers {2, 18, 4, 20, 19, 1, x, 21, 6}, what is the value of x?

(1) The median of the numbers is x.


(2) The mean of the numbers is between 12 and 13.

www.manhattanreview.com 19992016 Manhattan Review


Sets & Statistics Guide Questions 73

Solve yourself:

79. In a school, all students play at least one of the two games, rugby and baseball. What
percent of all students play only Rugby?

(1) 40% of all students play both rugby and baseball.


(2) 20% of the students who play baseball do not play rugby.

Solve yourself:

80. Among 40 employees in an office, how many own a vehicle?

(1) 60% of the employees are men.


(2) 30% of all women employees own a vehicle.

Solve yourself:

19992016 Manhattan Review www.manhattanreview.com


74 Sets & Statistics Guide Questions

Questions on Range

81. Seven different numbers are selected from the integers 1 to 100, and each number is
divided by 7. What is the sum of the remainders?
(1) The range of the seven remainders is 6.
(2) The seven numbers selected are consecutive integers.
Solve yourself:

82. Professor Vasquez gave a quiz, scored out of 100, to two classes, A and B. If in each class,
no two students received the same score and all scores were positive integers, was the
maximum score for class A greater than that for class B?
(1) In each class, the number of students taking the quiz was 25, and the lowest scores
in classes A and B were 66 and 76, respectively.
(2) The average (arithmetic mean) score on the quiz in classes A and B were 85 and 88,
respectively.
Solve yourself:

83. Does set X has all of its elements smaller than 15?
(1) Smallest element in the set is 2
(2) Range of set X is 13
Solve yourself:

84. A sequence is defined by a linear equation: 31 4x, where x is an integer greater than or
equal to 1. If a set is comprised of the first n terms of the sequence, what is the median
of the set?

www.manhattanreview.com 19992016 Manhattan Review


Sets & Statistics Guide Questions 75

(1) The sum of the terms of the set is 99.


(2) The range of the terms of the set is 32.

Solve yourself:

85. The mean of five distinct positive integers is 8. What is the value of the smallest number
in the set?

(1) The median of the set is 8.


(2) The range of the set is 6.

Solve yourself:

86. In the set of positive integers {3, x, 16, 11, 7, 2}, what is the value of x?

(1) The range of the numbers is x.


(2) The mean of the numbers is between 8 and 9.

Solve yourself:

19992016 Manhattan Review www.manhattanreview.com


76 Sets & Statistics Guide Questions

Questions on Standard deviation

87. If set L: {14, 13, 12, X, 10, 9, 8, Y, 6, 5, 4} has no two equal elements in the set, what is
the standard deviation of the set?

(1) X and Y are integers.


(2) Set L is arranged in descending order.

Solve yourself:

88. For set L: {14, 13, 12, X, 10, 9, 8, Y, 6, 5, 4}, what is the standard deviation?

(1) Set L is arranged in descending order.


(2) X Y >5

Solve yourself:

89. If set L: {14, 13, 12, X, 10, 9, 8, Y, 6, 5, 4} is arranged in descending order, what is the
standard deviation of the set?

(1) X and Y are integers.


(2) X Y <3

Solve yourself:

90. If Brian assembled 12 units of television sets, what is the standard deviation of number
of defects?

(1) Each of the first 4 television sets had 3 defects


(2) Range of number of defects in the last 9 television sets is 0.

www.manhattanreview.com 19992016 Manhattan Review


Sets & Statistics Guide Questions 77

Solve yourself:

91. If a set consists of multiples of 5, is the standard deviation of the set 0?

(1) Absolute values of all the elements are equal.


(2) All the elements of the set are equal.

Solve yourself:

92. Which of the sets, set X and set Y has a greater standard deviation?

(1) The range of set X is greater than the range of set Y.


(2) The average (arithmetic mean) of set X is greater than the average (arithmetic mean)
of set Y.

Solve yourself:

93. A set consists of consecutive odd integers. What is the standard deviation of the set?

(1) The set has 14 elements.


(2) The average (arithmetic mean) of the set is 7.

Solve yourself:

19992016 Manhattan Review www.manhattanreview.com


78 Sets & Statistics Guide Questions

94. Is the standard deviation of set A greater than the standard deviation of set B?

(1) Elements is set A are consecutive multiples of 5.


(2) Elements is set B are consecutive multiples of 3.

Solve yourself:

95. Each of 9 batches have a few number of defects. What is the standard deviation of the
numbers of defects in the 9 batches?

(1) The average (arithmetic mean) number of defects of the 9 batches is 3.


(2) Each of the 9 batches has the same number of defects.

Solve yourself:

96. Set Y is derived from taking 8 different integers from the set X given below. What is the
standard deviation of the numbers of set Y?

Set X: 3, 5, 7, 9, 11, 13, 15, 17, 19, 21

(1) The average (arithmetic mean) of the numbers of set Y is equal to the average (arith-
metic mean) of the numbers of set X.
(2) Sum of numbers not taken in the set Y is 24.

Solve yourself:

www.manhattanreview.com 19992016 Manhattan Review


Sets & Statistics Guide Questions 79

3.2.3 Data Interpretation

97. The bar-graph below shows the expenditure, in million dollars, on research by different
medical institutes combined together and the government, for different years. The data
for the expenditure made by the institutes in 2011 and the expenditure made by the
government in 2014 are not known.

In which year is the expenditure on research by the institutes as a percent of the expen-
diture by the government the highest?

(1) The growth rate in the expenditure from 2010 to 2011 by the institutes was 20%.

(2) The growth rate in the expenditure from 2014 to 2015 by the government was 25%.

Solve yourself:

98. The bar graph below shows the number of passengers using buses, railways and airlines
as means of transport in different years.

Passenger distribution
450
400
Millions of passengers

350
300
250
200
150
100
50
0
2000 2001 2002 2003

Bus Rail Airline

What is the percent increase in total revenue generated by buses, railways and airlines
combined from 2001 to 2002?

(1) The average airfare per passenger was $360 while those for buses and railways were
$3 and $90, respectively, in 2001.

(2) From 2001 to 2002, air fare increases by 20%, while the others remain constant.

19992016 Manhattan Review www.manhattanreview.com


80 Sets & Statistics Guide Questions

Solve yourself:

99. The pie-charts below show the proportion of revenue obtained from different sources of
a financial company.

2010 2011
Dividend
Dividend 5%
5% Others
Others 10%
20%
Leasing Leasing
45% Interest
25% 55%
Interest
20%

Securities Securities
10% 5%

If the total revenue earned in 2010 is $120 million, what is the percent change in revenue
earned by the company from Securities in 2012 over that in 2011?

(1) The total revenue earned in 2012 is 25 times than the revenue earned from Securi-
ties in 2011.

(2) Revenue earned from Securities in 2012 is 25% less than that in 2010.

Solve yourself:

100. The table below shows the distribution of members from different countries for a
Non-governmental organization (NGO).

www.manhattanreview.com 19992016 Manhattan Review


Sets & Statistics Guide Questions 81

Country Percent of members

India 55

USA x

Canada 5

UK 3

Others y

Is the percent contribution of the number of members by a country present in the region
denoted as Others greater than that by USA? It is known that the number of members
from each of the countries in the region Others is equal.

(1) The region Others consists of 6 countries.


(2) The percent contribution of USA is less than 10%.

Solve yourself:

19992016 Manhattan Review www.manhattanreview.com


82 Sets & Statistics GuideAnswer Key

www.manhattanreview.com 19992016 Manhattan Review


Chapter 4

Answer Key

83
84 Sets & Statistics GuideAnswer Key

4.1 Problem Solving Questions

(1) E (13) A (25) D (37) D (49) D

(2) E (14) E (26) A (38) B (50) A

(3) C (15) C (27) C (39) C (51) B

(4) D (16) A (28) A (40) C (52) A

(5) A (17) C (29) A (41) E (53) D

(6) E (18) E (30) C (42) D (54) A

(7) E (19) B (31) C (43) E (55) E

(8) D (20) B (32) B (44) D (56) B

(9) C (21) C (33) A (45) C (57) B

(10) D (22) D (34) C (46) E (58) D

(11) B (23) E (35) E (47) D (59) C

(12) C (24) D (36) B (48) E (60) A

www.manhattanreview.com 19992016 Manhattan Review


Sets & Statistics GuideAnswer Key 85

4.2 Data Sufficiency Questions

(61) C (69) A (77) E (85) B (93) A

(62) C (70) A (78) C (86) A (94) E

(63) C (71) C (79) C (87) C (95) B

(64) D (72) C (80) E (88) E (96) E

(65) A (73) E (81) B (89) C (97) C

(66) E (74) C (82) A (90) C (98) C

(67) E (75) C (83) C (91) B (99) E

(68) D (76) C (84) D (92) E (100) E

19992016 Manhattan Review www.manhattanreview.com


86 Sets & Statistics GuideAnswer Key

www.manhattanreview.com 19992016 Manhattan Review


Chapter 5

Solution

87
88 Sets & Statistics Guide Solutions

5.1 Problem Solving Questions

5.1.1 Sets

1. Since 53% of the total people did not take juice, we can say that (100 53) = 47% of the
total people took juice.

Let the number of people who took both be y.

Let us represent the above information using a Venn-diagram, as shown below:

One glass of juice One glass of wine


(47% of 100 = 47) (62% of 100 = 62)

47 62
% of 100 =
Only juice Only wine
Both

Neither (0)

We know that 53% did not take juice

=> 62 y = 53

=> y = 9

Thus, the final (solved) Venn-diagram is:

One glass of juice One glass of wine


(47) (62)

38 9 53

Only juice Both Only wine

Neither (0)

Thus, we have:
Number of glasses needed to serve those who took only juice = 38.

www.manhattanreview.com 19992016 Manhattan Review


Sets & Statistics Guide Solutions 89

Number of glasses needed to serve those who took only wine = 53.

Number of glasses needed to serve those who took both = 9 2 = 18.

Thus, total number of glasses used = 38 + 53 + 18 = 109.

Alternate approach:

First we calculate the value of those who took juice, i.e. 47.

Now, we observe that if we simply add the values of the two sets; i.e. those who took
juice, and those who took wine, their intersection would be added twice.

Since each glass is to be used only one, the people in the intersection would, actually
use 2 glasses each.

Thus, simply adding 47 and 62, we should get the total number of glasses.

Thus, total number of glasses used = 47 + 62 = 109.

The correct answer is option E.

2. Let us represent the above information using a Venn-diagram, as shown below:

Roof repairs Repainted


(36) (48)

20 (48 16 = 32)
(36 20 = 16)
Roof repairs but Repainted but
not Repainted Both no Roof repairs

Number of houses having both = 36 20 = 16.

Thus, the number of houses that were repainted but did not have roof repairs
= 48 16 = 32.

The correct answer is option E.

19992016 Manhattan Review www.manhattanreview.com


90 Sets & Statistics Guide Solutions

3. The above information can be represented in a Venn-diagram as shown below:

Physics Economics

0 Mathematics

Total = 66

Thus, we have:

28 students did not take up either Physics or Economics


=> c = 28 . . . (i)
15 students did not take up either Mathematics or Economics
=> a = 15 . . . (ii)
45 students did not take up Economics
=> a + c + e = 45
=> e = 45 (a + c) = 45 (28 + 15) => e = 2 . . . (iii)
30 students did not take up Mathematics
=> a + d + b = 30 => d + b = 30 a = 30 15 => d + b = 15 . . . (iv)
Number of students who did not take up either Mathematics or Economics, exceeded
the number of students who did not take up Mathematics or Physics by 2
=> a = b + 2 => b = a 2 = 15 2 => b = 13 . . . (v)
Thus, from (iv), we have: d = 15 b = 15 13 => d = 2 . . . (vi)
Total number of students is 66
=> a + b + c + d + e + f + 5 = 66
=> f = 66 (a + b + c + d + e + 5) = 66 (15 + 13 + 28 + 2 + 2 + 5)
=> f = 1
Thus, the number of students who took up both Economics and Mathematics
=f +5=1+5
=6

The complete Venn-diagram is shown below for reference:

www.manhattanreview.com 19992016 Manhattan Review


Sets & Statistics Guide Solutions 91

Physics Economics

15 2 13

5
2 1

28

0 Mathematics

Total = 66

The correct answer is option C.

4. The data can be represented in a Venn-diagram as shown below:

A (33) B (30)

5 C (17)

Total = 63

We know that:

A&B=B&C=A&C=9

Also, we know that:

A or B or C =63 5 = 58

Thus, we have:

A or B or C = A + B + C (A & B + B & C + A & C) +A & B & C

=> 58 = 33 + 30 + 17 (9 + 9 + 9) + x

=> 58 = 80 27 + x

=> x = 5

19992016 Manhattan Review www.manhattanreview.com


92 Sets & Statistics Guide Solutions

Alternate approach:

A (33) B (30)

33 9 9 9 30 9 9

= 15 + = 12 +

9 9

17 9 9
= 1

5 C (17)

Total = 63

Based on the Venn-diagram shown above, we have:


(15 + x) + (12 + x) + (x 1) + (9 x) + (9 x) + (9 x) + x + 5 = 63

=> 58 + x = 63

=> x = 5

The correct answer is option D.

5.

Factor Percent of respondents

User-friendly 56%

Fast response time 48%

Bargain prices 42%

Percent of respondents citing User friendly" = 56%

Percent of respondents citing Fast response time" = 48%

Percent of respondents citing both of the above = 30%

www.manhattanreview.com 19992016 Manhattan Review


Sets & Statistics Guide Solutions 93

For two sets A, B:

(A or B) = A + B (A and B)

Thus, percent of respondents citing User friendly" or Fast response time"


= 56% + 48% 30%

= 74%

The number of respondents who cited Bargain prices", but neither User-friendly" nor
Fast response time" will be the maximum if there is MINIMUM intersection (overlap)
between those who cited Bargain prices" and those who cited User-friendly or Fast
response time", also, the percent of respondents who did not cite any of the three should
be 0%, as shown in the diagram below:

Bargain prices User-friendly

We need to
maximize this
region

These three regions


(intersections) should
have a minimum value

0% Fast response time

The above diagram may be simplified to:

User-friendly OR
Bargain prices Fast response time
(42%) (74%)

We need to %
maximize this This region
region (intersection) should
have a minimum value

0%

Thus, we have:

42 + 74 x = 100

=> x = 16

Thus, the required maximum value


= (42% 16%) of 1200

19992016 Manhattan Review www.manhattanreview.com


94 Sets & Statistics Guide Solutions

= 26% of 1200

26
= 1200
100

= 312

The correct answer is option A.

6.
Pasta Burgers
63% 76%

Only Pasta Both Only Burgers


63 % % 76 %

Neither
%
Total = 100%

We have:

63 + 76 x + n = 100

=> x = n + 39 . . . (i)

Thus, the minimum value of x occurs when n = 0

=> x (Minimum) = 39

Again, the maximum value of x occurs when the entire 63% (Pasta) forms a part of 76%
(Burgers), thus, the maximum value of the intersection of Pasta and Burgers being 63%.

Thus, we have:

39 x 63 Statement I is correct

Percent of people who like only one among Pasta and Burgers

= (63 x) + (76 x) = 139 2x

The above is a maximum when x is minimum, i.e. 39

www.manhattanreview.com 19992016 Manhattan Review


Sets & Statistics Guide Solutions 95

Thus, the maximum percent of people who like only one among Pasta and Burgers

= 139 2 39 = 61 Statement II is correct

People who do not like either Pasta or Burgers

= n = x 39 . . . From (i)

The value of n is maximum when x is maximum, i.e. 63

Thus, the maximum percent of people who do not like either pasta or burgers

= 63 39 = 24 Statement III is correct

Thus, all three statements are correct.

The correct answer is option E.

7. The three rides are: Merry-go-round (M), Roller-coaster (R) and Toy-train (T).

We have:

d + e + f = 10

We need to determine the total cost of all rides. Thus, we need to know the total number
of rides.

From the diagram, we have:

M R
11 14


T
=0

Total children = 30

19992016 Manhattan Review www.manhattanreview.com


96 Sets & Statistics Guide Solutions

(a + b + c) children take one ride each, (d + e + f ) children take 2 rides each, and 2
children take three rides each.

Thus, total number of rides

= (a + b + c) 1 + (d + e + f ) 2 + 2 3

= (a + b + c + d + e + f + 2) + (d + e + f ) + 4

= 30 + 10 + 4 = 44

Thus, the total cost of all rides

= $ (5 44) = $220

The correct answer is option E.

8.
Fail English Fail Mathematics
36% 30%

Fail
Both
%

Pass Both
%
Total = 100%

Let us first maximize the value of n.

The value of n is maximized if all of those who fail in Mathematics (30%) forms a part of
those who fail in English (36%).

Thus, n = 100 36 = 64% (Maximum)

Let us now minimize the value of n.

The value of n is minimized if there is no overlap between those who fail in Mathematics
and those who fail in English.

Thus, n = 100 (36 + 30) = 34% (Minimum)

www.manhattanreview.com 19992016 Manhattan Review


Sets & Statistics Guide Solutions 97

Thus, the required value = 64% + 34% = 98%

The correct answer is option D.

9.
A B
30% 20%

85%
= 5% C
Total = 100%

We have: d + e + f = 20

For three sets A, B, C, we have:

A or B or C = (A + B + C) ((A and B) + (B and C) + (C and A)) +(A and B and C)

  
=> 100 5 = 30 + 20 + 85 d+g + f +g + e+g +g


=> 95 = 135 (d + e + f ) + 3g + g


=> 95 = 135 20 + 3g + g

=> g = 10

The correct answer is option C.

19992016 Manhattan Review www.manhattanreview.com


98 Sets & Statistics Guide Solutions

5.1.2 Statistics

Questions on Average (Arithmetic Mean)

10. If all the elements of a set are multiplied by a constant, the mean also gets multiplied by
the same constant.

However the mean remaining unchanged can happen only and only if mean = 0, so the
statement I is true.

Since sum of all the elements = Mean # of elements


=> Sum of all the elements = 0 as mean = 0, so the statement II is true.

Statement 3 is also not necessary because the set may contain all zeros.

The correct answer is option D.

11. We know that arithmetic mean of x and y is 80,


=> (x + y)/2 = 80
=> x + y = 160 (1)

Also, we know that arithmetic mean of x and z is 200,


=> (x + z)/2 = 200
=> x + z = 400 (2)

To get the value of (z y)/8, we subtract equation (2) from the equation (1) and get,
=> z y = 400 160 = 240;
=> (z y)/8 = 240/8 = 30

The correct answer is option B.

12. Say the three numbers are a, 7, and b; a being the largest and b being the smallest.

=> (a + 7)/2 = 8
=> a + 7 = 16
=> a = 9

Similarly, (a + 7 + b)/3 = 6

=> 9 + 7 + b = 18
=> b = 2

So, the numbers are: 9, 7, & 2

www.manhattanreview.com 19992016 Manhattan Review


Sets & Statistics Guide Solutions 99

So, the mean of the two smallest numbers = (7 + 2)/2 = 4.5

The correct answer is option C.

13. Given,

Average = 7.5

=> Sum of 6 numbers = 6average = 67.5 = 45

Say the number multiplied by 4 is x, so instead of x, the number becomes 4x. Or the
sum of the numbers increases by 3x.

=> New sum = 45 + 3x

Given that new average = 7.5 0.5 = 7


=> New sum = 6 new average = 67 = 42
=> 45 + 3x = 42
=> x = 3

The correct answer is option A.

Note that the question stem stated that after multiplying a number by 4, the average de-
creased rather than anticipatedincreased, it implies that the number must be negative,
thus only options A or B could be true.

14. Sum of scores of 5 tests = 5 38 = 190;

Say the score on the 6th test is x, so

Sum of scores of 6 tests = 190 + x;

(190 + x)
=> Average score = ;
6
(190 + x)
=> 41 = ; given that the average is 41.
6

=> x = 56

The correct answer is option E.

Alternate approach:

19992016 Manhattan Review www.manhattanreview.com


100 Sets & Statistics Guide Solutions

You find that to increase the average score of 38 to 41, you need to increase it by 3.
Since 3 is an average increase per test, so to get the total increase, you must multiply it
by the total number of tests i.e. 6.

=> Total increase = 36 = 18;

=> Desired score in the 6th test = 38 + 18 = 56

15. Total rainfall in first x days = 32x;

Total rainfall in (x + 1) days = (32x + 56);

(32x + 56)
=> Average rainfall in (x + 1) days =
(x + 1)
(32x + 56)
=> 36 = ; given that average rainfall in (x + 1) days = 36 mm
(x + 1)

=> 36(x + 1) = (32x + 56)

=> 36x + 36 = 32x + 56

=> 4x = 20

=> x = 5 days

The correct answer is option C.

Alternate approach:

Given that increase in average = 36 32 = 4;

And difference of average of x days, 32 and todays rainfall, 56 = 56 32 = 24;

=> 24 = 4 (x + 1)

=> x = 5 days

16. Let the minimum score obtained in any test be m

Thus, mean score = (m + 70)

Also, median score = (m + 50)

www.manhattanreview.com 19992016 Manhattan Review


Sets & Statistics Guide Solutions 101

Since the range is 160, the maximum score = (m + 160)

Since the number of tests is an odd number, there must be one test having the median
score, i.e. (m + 50)

Thus, we have three scores with us: m, (m + 50) and (m + 160)

m + m + 50 + m + 160 3m + 210
 
The mean of the above three scores is = = m + 70
3 3

Since the mean mentioned is also (m + 70) and matches with the derived mean, the
minimum number of tests is 3.

The correct answer is option A.

17. Since the average of 7 numbers is 12, the sum of the numbers = 7 12 = 84.

Let the numbers, when arranged in ascending order, be a, b, c, d, e, f and g.

Thus, we have:

a + b + c + d + e + f + g = 84 . . . (i)

Since the average of the 4 smallest numbers in this set is 8, we have:

a + b + c + d = 8 4 = 32 . . . (ii)

Since the average of the 4 greatest numbers in this set is 20, we have:

d + e + f + g = 20 4 = 80 . . . (iii)

(ii) + (iii) (i):

d = 28

=> a + b + c = 32 28 = 4

e + f + g = 80 28 = 52

Thus, the required difference = 52 4 = 48

The correct answer is option C.

19992016 Manhattan Review www.manhattanreview.com


102 Sets & Statistics Guide Solutions

18. Let the number of members be x.

Thus, the sum of ages of all the members = 32x

We know that the sum of ages of 4 members = 25 4 = 100

Thus, the sum of ages of (x 4) members = 32x 100

32x 100
=> the average of (x 4) =
x4
32x 100
=> 60; We know that no member is more than 60 years of age
x4
=> 32x 100 60x 240
=> x 5
=> x can assume any value 5, 6, 7, 8, 9, . . .

The correct answer is option E.

Alternate approach:

Let us use the options to determine the number of members in the club.

Option A: 5 members:

Sum of ages of 5 members = 325 = 160


Since the average age of 4 members is 25 years, their total age = 25 4 = 100 years
Thus, the age of the 5th member = 160 100 = 60 60 (maximum possible age)
Satisfies

Option B: 6 members:

Sum of ages of 6 members = 326 = 192


Since the average age of 4 members is 25 years, their total age = 25 4 = 100 years
Thus, the sum of ages of 2 members = 192 100 = 92
92
Thus, the average age of 2 members = = 46 60 (maximum possible age)
2
Satisfies

Option C: 9 members:

Sum of ages of 9 members = 329 = 288


Since the average age of 4 members is 25 years, their total age = 25 4 = 100 years
Thus, the sum of ages of 5 members = 288 100 = 188

www.manhattanreview.com 19992016 Manhattan Review


Sets & Statistics Guide Solutions 103

188
Thus, the average age of 5 members = = 33.60 60 (maximum possible age)
5
Satisfies

19. Since the average of 5 integers is 65, the total = 65 5 = 325

The largest integer is 75

Thus, sum of the remaining 4 integers = 325 75 = 250

250
Thus, the average of the 4 integers would be = = 62.5
4

The smallest integer will be maximized if these 4 integers are as close to each other as
possible.

Since the numbers are distinct integers, let us subtract 0.5 from the average (62.5) for
one of the numbers and add 0.5 to the average (62.5) for the other number; giving us
62.5 0.5 = 62 and 62.5 + 0.5 = 63.

There are still two numbers left.

Let us subtract 1.5 from the average (62.5) for one of them and add 1.5 to the average
(62.5) for the other; giving us 62.5 1.5 = 61 and 62.5 + 1.5 = 64.

Thus, the 4 integers are 61, 62, 63 and 64.

Thus, the maximum possible value of the smallest integer = 61.

Alternate approach:

Since we need to maximize the smallest number, we need to make the distance between
the consecutive numbers minimum, in this scenario 1.

Let the smallest number be x, thus the 2nd smallest number = (x + 1), and other
numbers (x + 2) and (x + 3).

We have the sum of the above 4 numbers as 250.

x + (x + 1) + (x + 2) + (x + 3) = 250

=> 4x = 244

19992016 Manhattan Review www.manhattanreview.com


104 Sets & Statistics Guide Solutions

=> x = 61

The correct answer is option B.

20. Since the average contribution by 4 students was $20, total contribution = $(20 4) = $80

To maximize the ratio of the amounts contributed by any two students, we need to max-
imize the contribution of one student and minimize the contribution by another student.

To minimize the contribution by any student, we need to maximize the contribution


made by the other students. Since the contributions are different, we assume that the
contributions made by 3 students are $25, $24 and $23.

Thus, contribution (minimum) made by the 4th student = $(80 25 24 23) = $8

25 1
Thus, the required ratio = =3
8 8

The correct answer is option B.

21. Let the n terms be t1 , t2 , t3 , . . . tn

Since the mean is x, we have:

t1 + t2 + + tn
=x
n

=> t1 + t2 + + tn = nx

The new series is obtained by adding 1 to the 1st term, 2 to the 2nd term, and so on.

Thus, the new terms are:

(t1 + 1) , (t2 + 2) , (t3 + 3) , . . . (tn + n)

Thus, the sum of the above terms

= t1 + t2 + + tn + (1 + 2 + 3 + + n)

n (n + 1)
= nx +
2

Thus, the new mean

www.manhattanreview.com 19992016 Manhattan Review


Sets & Statistics Guide Solutions 105

n (n + 1)
nx +
= 2
n
n+1
=x+
2

The correct answer is option C.

Alternate Approach:

Let us assume that there are three terms (1, 2, 3) and their mean = 2, thus n = 3 & x = 2

After the increment, the terms are: 1 + 1, 2 + 2, 3 + 3 => 2, 4, 6

2+4+6
The new average = =4
3

By plugging the values of x & n in options, we find that both option C and D give the new
average = 4. So we cannot decide which is the correct answer. Sometimes test-makers
make such tricky options so that if one goes for a hit and trial approach, he or she may
still not be able to conclude the correct answer.

Since we have zeroed in on options C and D, we need to eliminate one of them, but for
that to happen, we will have to choose another set of numbers.

Let us assume that there are three terms (2, 3, 4) and their mean = 3, thus n = 3 & x = 3

After the increment, the terms are: 2 + 1, 3 + 2, 4 + 3 => 3, 5, 7

3+5+7
The new average = =5
3

We can still not eliminate C or D as both the options still qualify.

Let us take another set:

Let us assume that there are four terms (1, 3, 5, 7) and their mean = 4, thus n = 4 & x = 4

After the increment, the terms are: 1 + 1, 3 + 2, 5 + 3, 7 + 4 => 2, 5, 8, 11

2 + 5 + 8 + 11
The new average = = 6.5
4

Let us plug in x = 4 & n = 4 in options C and D.

19992016 Manhattan Review www.manhattanreview.com


106 Sets & Statistics Guide Solutions

n+1 4+1
(C) x+ =4+ = 6.50 Satisfies
2 2
(D) x + n 1 = 4 + 4 1 = 7 6= 6.50 Does not satisfy

www.manhattanreview.com 19992016 Manhattan Review


Sets & Statistics Guide Solutions 107

Questions on Median

22. Number of defects in the first 6 cars = 9, 7, 10, 4, 6 and n, respectively.

Thus, the total number of defects = (9 + 7 + 10 + 4 + 6 + n) = 36 + n

Thus, the mean number of defects


36 + n
=
6

We know that the mean number of defects and the median number of defects are the
same.

Working with the statements:

I. n = 3:
36 + 3
Mean = = 6.5
6

The number of defects, when arranged in order, is: 3, 4, 6, 7, 9 and 10

The median is the average of the two middle terms.

Thus, we have:

6 th
(  th )
6

term+ + 1 term
n o
2 2 3rd term+4th term
Median = =
2 2
6+7
= = 6.5
2

Thus, Mean = Median Satisfies

II. n = 7:
36 + 7
Mean = = 7.16
6

The number of defects, when arranged in order, is: 4, 6, 7, 7, 9 and 10

The median is the average of the two middle terms.

Thus, we have:

6 th
(  th )
6

term+ + 1 term
n o
2 2 3rd term+4th term
Median = =
2 2

19992016 Manhattan Review www.manhattanreview.com


108 Sets & Statistics Guide Solutions

7+7
= =7
2

Thus, Mean 6= Median Does not satisfy

III. n = 12:

36 + 12
Mean = =8
6

The number of defects, when arranged in order, is: 4, 6, 7, 9, 10 and 12

The median is the average of the two middle terms.

Thus, we have:

6 th
(  th )
6

term + + 1 term
n o
2 2 3rd term + 4th term
Median = =
2 2
7+9
= =8
2

Thus, Mean = Median Satisfies

The correct answer is option D.

23. Let us first arrange the elements of the set in ascending order, ignoring p as we do not
yet know where it lies.

=> 26, 2, 1, 3, 4, 16, 18

It is not necessary to know the exact placement of p in order to find which option could
be the median of the set.

When p is included, the set has an even number of elements. Consequently, the median
will be the average of the two middle-most terms.

It is important to note that the set consists of "unique integers". So, p cannot be equal to
any of the existing term of the set. This analysis itself answers the question. The correct
answer is option E: 3, as 3 is one of the terms of the set. However for the sake of better
understanding, we deep dive the options.

Let us analyze the set on the basis of the placement of p.


(1) Say p < 26, so the set would be: {p, 26, 2, 1, 3, 4, 16, 18}
=> Median = (1 + 3)/2 = 1

www.manhattanreview.com 19992016 Manhattan Review


Sets & Statistics Guide Solutions 109

(2) Say 2 > p > 26, so the set would be: {26, p, 2, 1, 3, 4, 16, 18}
=> Median = (1 + 3)/2 = 1 (No change in the value of median)

(3) Say 3 > p > 1, so the set would be: {26, 2, 1, p, 3, 4, 16, 18}; p cannot lie
between 2, and 1 as p is an unique integer.
=> Median = (p + 3)/2;

(a) p can have any value: 0, 1, or 2

(b) Median = (0 + 3)/2 = 1.5, or (1 + 3)/2 = 2, (2 + 3)/2 = 2.5

(4) Say 4 > p, so the set would be: {26, 2, 1, 3, 4, p, 16, 18} or
{26, 2, 1, 3, 4, 16, p, 18} or {26, 2, 1, 3, 4, 16, 18, p}

=> In either case, Median = (4 + 3)/2 = 3.5

So the possible values of median are: 1, 1.5, 2, 2.5, & 3.5.

The correct answer is option E.

24. Let us calculate the median for the given set.

First, arrange the set in ascending order: {7, 13, 18, 21, 22, 28}

=> Median = (18 + 21)/2 = 19.5

Desired median = 19.5 1.5 = 18

Since upon including an extra number, the number of elements in the set would be 7, an
odd number, and to make sure that 18 (the desired median) lies in the middle, we can
only add a new number that is 18.

The new set may look like the following.

#, 7, 13, 18, 21, 22, 28 or 7, #, 13, 18, 21, 22, 28 or 7, 13, #, 18, 21, 22, 28. In each case,
median is 18.

The correct answer is option D.

25. First, arrange the set X in an ascending order, we get: 8, 2, 4, 6, 23

19992016 Manhattan Review www.manhattanreview.com


110 Sets & Statistics Guide Solutions

=> Median = 4 (Middle-most term)


=> Median of set Y = 4

Arranging set Y in ascending order, we get: 8, 2, 0, 8, 12, 13; excluding y as we do not


know its value. We know that set Y has an odd number of terms, so there would be a
unique term, equaling median value (here 4). Since there is no term, equaling 4, hence
only y can be 4.

The correct answer is option D.

26. Median of all consecutive integers being a non-integer implies that the number of
consecutive integers, X is an even number.

The median, 50.5 is derived by taking the average of 50 & 51.

The list looks like this,


N1, N2, N3, ......, 50, 51, ......;

Since X is even, X/2 integers will be on the left of the median, and X/2 integers will be
on the right of the median, 50.5. Counting of the smaller integer must start from 51 and
we must deduct X/2 to reach the smallest, thus the smallest integer = 51 X/2.

The correct answer is option A.

Alternate approach:

Since we already concluded that X is an even number, we can assume X.

Say X = 4. So, the list would be:


49, 50, 51, 52; smallest being 49
Only option A, 51 X/2 = 51 4/2 = 49 is applicable.

27. The numbers of marbles with the 5 people are: 23, 45, 51, 66 and 73

Thus, total number of marbles = 23 + 45 + 51 + 66 + 73 = 258.

We need to maximize the median value.

th
5+1

The median of 5 terms is the term = 3rd term.
2

Let the terms be arranged in ascending order.

www.manhattanreview.com 19992016 Manhattan Review


Sets & Statistics Guide Solutions 111

Since the median has to be maximized, we need to maximize the terms following the
median. Thus, we need to maximize the 3rd , 4th and 5th terms.

Thus, we minimize the 1st and 2nd terms, keeping them at 1 and 2, respectively.

Thus, the number of marbles with the other 3 people = 258 1 2 = 255.

Since the above 3 people have different numbers of marbles, let the numbers be
a, (a + 1) and (a + 2).

Thus, we have:

a + (a + 1) + (a + 2) = 255 => a = 84

Thus, the numbers of marbles are: 1, 2, 84, 85 and 86 (the median is maximized).

Thus, the range = 86 1 = 85

The correct answer is option C.

28. Total number of students = 5x


Since boys and girls are in the ration 2 : 3, we have:

2
Number of boys = 5x = 2x
2+3

Number of girls = 5x 2x = 3x

Average height of boys = (3x + 1) feet.

Thus, sum of heights of all boys = 2x (3x + 1) = 6x 2 + 2x

Average height of girls = (6 2x) feet.

Thus, sum of heights of all girls = 3x (6 2x) = 18x 6x 2

Thus, sum of heights of all students


= Sum of heights of boys + Sum of heights of girls = 20x

20x
Thus, the average height of all students = = 4 feet
5x

The correct answer is option A.

19992016 Manhattan Review www.manhattanreview.com


112 Sets & Statistics Guide Solutions

29. Total number of students = 5x


Since boys and girls are in the ration 2 : 3, we have:

2
Number of boys = 5x = 2x
2+3

Number of girls = 5x 2x = 3x

Average height of boys = (3x + 1) feet.

Thus, sum of heights of all boys = 2x (3x + 1) = 6x 2 + 2x

Average height of girls = (6 2x) feet.

Thus, sum of heights of all girls = 3x (6 2x) = 18x 6x 2

Thus, sum of heights of all students


= Sum of heights of boys + Sum of heights of girls = 20x

20x
Thus, the average height of all students = = 4 feet
5x

The correct answer is option A.

30. In the table given above, the median salary of each age-group is considered, i.e. the
salary of one individual is considered as a representation of each age-group.

Thus, the mean salary is simply the sum of the above 5 salaries, divided by 5.

th
5+1

Similarly, the median salary is the term, i.e. 3rd term, when the above salaries
2
are arranged in order.

Thus, we have:

12000 + 24000 + 36000 + 38000 + 28000


 
Mean = $ = $27600
5

Median: The salaries in ascending order are: 12000, 24000, 28000, 36000 and 38000:

=> Median = $28000

After the two age groups are ignored, we have:

www.manhattanreview.com 19992016 Manhattan Review


Sets & Statistics Guide Solutions 113

24000 + 36000 + 38000


 
Mean = $ = $32667
5

Median: The salaries in ascending order are: 24000, 36000 and 38000:

=> Median = Middle term = $36000

Thus, we have:

Difference in mean = $32667 $27600 = $5067 Statement I is incorrect

Difference in median = $36000 $28000 = $8000 Statement II is incorrect

Difference between new mean and median = $36000 $32667 = $3333 - Statement III is
correct

Thus, only Statement III is correct.

The correct answer is option C.

31. We have:

3x 5y
3x1 5y =
3
3x 5y
3x 5y1 =
5
3x 5y
Thus, the smallest of the three terms is and the largest is 3x 5y
5
3x 5y 3x 5y
Thus, the three terms, when arranged in ascending order are: , and 3x 5y
5 3
3x 5y
Thus, the median value = = 3x1 5y
3

Thus, we have:

3x1 5y = 2025 = 34 52

=> x 1 = 4, i.e. x = 5 and y = 2

=> xy = 10

The correct answer is option C.

19992016 Manhattan Review www.manhattanreview.com


114 Sets & Statistics Guide Solutions

Questions on Average and Median

32. Since the average of the five positive integers is 16, we have:

k+m+r +s+t
= 16
5
=> k + m + r + s + t = 80

Since t = 40, we have:

k + m + r + s = 80 40 = 40 . . . (i)

Also, we have:
k<m<r <s<t

=> k < m < r < s < 40

The median is the middle term when the numbers are arranged in order.

Thus, the median is r .

Since we need to maximize r , and at the same time keep all the numbers positive integers,
we have:
k = 1, m = 2

Thus, from (i), we have:


r + s = 40 (1 + 2) = 37

Since r < s, we have:


s = (r + 1)

r = 18, s = 19

Thus, the maximum value of the median is 18.

The correct answer is option B.

33. We know that the average number of correct answers was 50 and the median number of
correct answers was 40.

www.manhattanreview.com 19992016 Manhattan Review


Sets & Statistics Guide Solutions 115

th
9+1

Since there were 9 candidates, the median value would be that for the = 5th
2
student (after arranging in ascending or descending order).

Let the average of the number of correct answers of the first 4 students be f , and the
average of the number of correct answers of the last 4 students be l.

Since the average number of correct answers is 50, we have:


4f + 40 + 4l
= 50
9

=> 4f + 4l = 410

=> f + l = 102.5 . . . (i)

Since the number of correct answers of the first four students must be less than or equal
to 40, we have:

f 40

Thus, from (i):

l 102.5 40 = 62.5

Thus, the average number of correct answers of the last 4 students is at least 62.5

Thus, at least one student must have got more than 60 correct answers.

Thus, statement I is definitely correct.

However, the other statements may or may not be correct.

The correct answer is option A.

34. Given,

Average wt. of 9 boys = 36 kg;


=> Total wt. of 9 boys = 936 = 324 kg

Median wt. of 9 boys = 38 kg


Say the wt. of the lightest boy is x kg
=> Wt. of the heaviest boy = (3x 3) kg

19992016 Manhattan Review www.manhattanreview.com


116 Sets & Statistics Guide Solutions

Since the # of boys is an odd number, a boy with median wt., 38 kg, must be present
among 9 boys. As of now we have some information about only 3 boys. If we arrange
the boys according to their weights, they would be put up as following:

x, B2, B3, B4, 38, B6, B7, B8, (3x 3)


=> x = Wt of the lightest boy (B1); 38 = Median (B5); & (3x3) = Wt of the heaviest boy (B9)

Since the question asks us to get the maximum value of (3x 3), we must try to keep
the weights of other 6 boys to the lowest.

=> Since B1 = x is the lowest value, the least value for each of B2, B3, & B4 would be x
=> Similarly, Since Median = B5 = 38, the least value for each of B6, B7, & B8 would be 38

As per the designated values, the arrangement of boys according to their weights is
given below.

x, x, x, x, 38, 38, 38, 38, (3x 3)

=> Sum of 9 boys = x + x + x + x + 38 + 38 + 38 + 38 + (3x 3)


=> 324 = 7x + 152 3
=> 7x = 175
=> x = 25 kg
=> Wt of the heaviest boy = 3x 3 = 3 25 25 = 72 kg

The correct answer is option C.

35. Given that,

Median defect per batch = Average defect per batch; for all the nine batches;

Let us first deduce the median.

By arranging the eight values of defects in ascending order, we get;

2, 3, 3, 4, 5, 6, 7, 8

From the options, if the # of defects in the ninth batch is 2, 3, or 4, the median would be
4Middle-most value.

However if it is either 5, 6, 7 or 8, the median would be 5Middle-most value, so we are


not sure about the value of median, but it would be either 4 or 5.

www.manhattanreview.com 19992016 Manhattan Review


Sets & Statistics Guide Solutions 117

Let us calculate the average now. It must be either 4 or 5 as it is given that average =
median.

Say the # of defects in the ninth batch is x,


=> Average = (2 + 3 + 3 + 4 + 5 + 6 + 7 + 8 + x)/9 = (38 + x)/9;
=> (36 + 2 + x)/9
=> 4 + (2+x)
9

(2+x)
9 must be 0 or 1, thus x must be 2 or 7.

Since # of defects cannot be negative, so x = 7.

The correct answer is option E.

36. The sum of the numbers of the set

 
= x + y + x + y + x 4y + xy + 2y


= 3x + xy = x y + 3

Thus, the mean



x y +3
=
6

Since the mean is y + 3 , we have:

x y +3
=y +3
6

=> x = 6

We know that: y > 6

=> y = 6 + k, where k is a positive number

  
Thus, the terms are: x, y, x + y , x 4y , xy, 2y

= {6, (6 + k) , (12 + k) , (18 4k) , (36 + 6k) , (12 + 2k)}

Arranging in ascending order, we have:

{(18 4k) , 6, (6 + k) , (12 + k) , (12 + 2k) , (36 + 6k)}

19992016 Manhattan Review www.manhattanreview.com


118 Sets & Statistics Guide Solutions

Thus, the median of the above 6 terms is the average of the 3rd and 4th terms

(6 + k) + (12 + k)
=
2

= 9 + k = 3 + (6 + k)

6
=3+y = +y
2
x
= +y
2

Alternate approach:

We have determined that x = 6.

Since y > 6 => y > x

Thus, the terms in ascending order are:

  
x 4y , x, y, x + y , 2y, xy

y + x+y x
Thus, the median is the average of the 3rd and 4th terms = = +y
2 2

The correct answer is option B.

37. Since the mean of the 4 numbers is 32, we have:

a+b+c+d
= 12
4

=> a + b + c + d = 48 . . . (i)

Since the median is 7, and the numbers are in ascending order, we have:

b+c
=7
2

=> b + c = 14 . . . (ii)

We know that one of the numbers is 10. Thus, we have the following possibilities:

a = 8: Not possible, since the mean of b and c, both of which are greater than a, is
7

www.manhattanreview.com 19992016 Manhattan Review


Sets & Statistics Guide Solutions 119

b = 8: Not possible, since b + c = 14 and b < c


c = 8: This is possible, since from b + c = 14, thus, we have b = 6.

Thus, to minimize d, we need to maximize a. The maximum value of a can only be 5


since a < b.

Thus, we have: d = 48 (a + b + c) = 48 (5 + 14) = 29

The correct answer is option D.

38. Mean of the three terms

3x1 + 3x1 5 + 3x 52
=
3

3x1 1 + 5 + 3 52

=
3

= 3x2 (1 + 5 + 75)

= 3x2 34

= 3x+2

Thus, we have:

3x+2 = 243 = 35

=> x = 3

Thus, the three terms are: 3x1 , 3x1 5 and 3x 52

= 331 , 331 5 and 33 52

Thus, the median is 32 5 = 45.

The correct answer is option B.

19992016 Manhattan Review www.manhattanreview.com


120 Sets & Statistics Guide Solutions

Questions on Range

39. We do not know the values of a, b, & c. So,

Minimum value of range = 76 (43) = 119; if any of a, b, & c is greater than 76 and/or
any of a, b, & c is smaller than 43, the range would be greater than 119.

Since range is always +ive, so option I is ruled out.

Option II is ruled out as its value is less than 119, we know that the minimum value of
range should be 119.

The correct answer is option C.

40. The smallest 3 numbers are reduced by 2 each, thus, the total is reduced by 3 2 = 6.

The largest 2 numbers are increased by 3 each, thus, the total is increased by 2 3 = 6.

Thus, the sum of the 8 numbers of the set remains unchanged, as the sum is reduced
and increased by the same number.

Thus, the mean remains unchanged. Statement I is incorrect

The median of the set of 8 numbers is the average of the 4th and 5th numbers, after the
numbers are arranged in order, say, ascending order.

The smallest three numbers, i.e. 1st , 2nd and 3rd numbers are reduced, and the largest
two numbers, i.e. 7th and 8th numbers are increased.

Thus, the 4th and 5th numbers remain unchanged. Also, the order of the numbers
remains the same since the smaller numbers become smaller and the larger numbers
become larger.

Thus, the median remains unchanged. Statement II is incorrect

The range is the difference between the largest and the smallest numbers. Since the
largest number is increased by 3 and the smallest number is reduced by 2, the range will
increase by 3 (2) = 5.

Thus, the range would change. Statement III is correct.

The correct answer is option C.

www.manhattanreview.com 19992016 Manhattan Review


Sets & Statistics Guide Solutions 121

41. Since the range of heights of students in Class A is 9 inches, let the height of the shortest
and the tallest students in Class A be x and (x + 9) inches, respectively.

Since the range of heights of students in Class B is 6 inches, let the height of the shortest
and the tallest students in Class B be y and y + 6 inches, respectively.

We need to determine the range of heights of all students taken together. Thus, we need
to know the relative values of x and y.

Statement I: The tallest student in Class B is 5 inches taller than the tallest student
in Class A

=> y + 6 = (x + 9) + 5 => y = x + 8

=> The heights of the shortest and the tallest students of Class B are y = x + 8
and y + 6 = x + 14, respectively

The heights of the shortest and the tallest students of Class A are x and x + 9,
respectively

Thus, considering all students, the height of the shortest student is x inches and
the height of the tallest student is (x + 14) inches.

Thus, the range is 14 inches.

Statement II: The shortest student in Class B is 1 inches shorter than the tallest
student in Class A

=> y = (x + 9) 1 => y = x + 8

=> The heights of the shortest and the tallest students of Class B are y = x + 8
and y + 6 = x + 14, respectively

The heights of the shortest and the tallest students of Class A are x and x + 9,
respectively

Thus, considering all students, the height of the shortest student is x inches and
the height of the tallest student is (x + 14) inches.

Thus, the range is 14 inches.

Statement III: The shortest student in Class A is 8 inches shorter than the shortest
student in Class B

19992016 Manhattan Review www.manhattanreview.com


122 Sets & Statistics Guide Solutions

=> x = y 8 => y = x + 8

=> The heights of the shortest and the tallest students of Class B are y = x + 8
and y + 6 = x + 14, respectively

The heights of the shortest and the tallest students of Class A are x and x + 9,
respectively

Thus, considering all students, the height of the shortest student is (x) inches and
the height of the tallest student is (x + 14) inches.

Thus, the range is 14 inches.

Thus, the range of all students combined can be determined from each of the statements.

The correct answer is option E.

www.manhattanreview.com 19992016 Manhattan Review


Sets & Statistics Guide Solutions 123

Questions on Standard deviation

42. Let us analyze the situations of John and Bob:

John:

Johns score = 75

Mean score of all students = 70

Difference between Johns score and the mean score = 75 70 = 5

Standard deviation of all scores = 2

5
Thus, number of Standard deviations that Johns score is more than the mean score =
2

= 2.5

Bob:

Bobs score = 72

Mean score of all students = 65

Difference between Bobs score and the mean score = 72 65 = 7

Standard deviation of all scores = 3

Thus, number of Standard deviations that Bobs score is more than the mean score
7
=
3
= 2.33

Thus, Johns score is farther away from the mean than is Bobs score.

Thus, Johns performance in his exam is better than that of Bob. Statement I is correct

Considering statement II:

Since Johns score is higher than the mean score, there may or may not be a student who
has scored higher than what John has scored. Statement II is incorrect

19992016 Manhattan Review www.manhattanreview.com


124 Sets & Statistics Guide Solutions

Considering statement III:

Since Bobs score is higher than the mean score, there must be a student who has scored
lesser than the mean score, and hence, has scored lesser than what Bob has scored.
Statement III is correct.

The correct answer is option D.

43. Adding or subtracting a constant number from all the elements of a set does not affect
the standard deviation (SD) of the set. We see that all the setP, Q and R can be obtained
by adding or subtracting a constant from the elements of the given set X, hence all the
sets have equal standard deviations (SD).

The correct answer is option E.

Note that, if you multiply each element of a set by a constant m, the SD would be m
times the original SD. Say, each element of the set is multiplied by 3, then the SD would
be 3 times the original SD = 3SD.

Similarly, if you divide each element of the set by a constant m, the SD would be 1/m
times the original SD = SD/3.

Multiplying with or dividing by a negative number does not affect SDs sign. SD is always
a positive number.

44. Adding or subtracting a constant from each element in the set has no effect on standard
deviation; however multiplication and division does change SD.

Looking at options A, B, C & E, we find that a constant term is NOT added to or NOT
subtracted from each element, this implies that SD of any of the set would not be the
same as the SD of set X. So only option left is: Dcorrect answer.

Though at first sight, it does not look that option D satisfies the criterion, expanding
each element will do the trick.

D. {p + qr , q(1 + r ), r (q + 1)}

I element: p + qr
II element: q(1 + r ) = q + qr
III element: r (q + 1) = r + qr

You must have observed that each element is derived by adding a constant qr 0 to each
element of set X. This implies that SD of the set would be the same as that of set X.

www.manhattanreview.com 19992016 Manhattan Review


Sets & Statistics Guide Solutions 125

Hence, the correct option is D.

Alternate Approach:

Say, p = 1, q = 2& r = 3. No need to calculate SD.

Let us analyze the options by plugging in the values of p, q& r . The values would be as
follows.

A. 2, 2, 3
B. 1, 3, 3
C. 1, 2, 4
D. 7, 8, 9: Correct; as constant 6 is added to {p = 1, q = 2, r = 3}, implying same SD.
E. 0, 2, 3

45. We know that adding or subtracting a constant number (here, 100) from all the elements
of a set does not affect the standard deviation(SD) of the set. So the question boils down
to

"Which of the following represents the correct ordering of the sets in terms of the absolute
increase in their Standard Deviation, from the largest to the smallest?"

It is to be noted that the number of elements in each is the same, 5 each. so only the
deviations w.r.t. mean would matter and be deciding.

For set A, we find that the deviation of any element from the successive element is 10.
For set B, we find that the deviation of any element from the successive element is 5.
For set C, we find that the deviation of any element from the successive element is 20.

There is no need to calculate SDs for sets, we can conclude that SD for set C would be
largest and SD for set B would be smallest.

Note that above logical deduction approach is applicable if there are


equal number of elements in each set, and elements are evenly spaced.

The correct answer is option C.

46. Since SD is always +ive, so option I & II are ruled out.

SD for the given set CANNOT be 0 as there are deviations among elements. For SD to
be 0, all the elements of a set must be equal.

The correct answer is option E.

19992016 Manhattan Review www.manhattanreview.com


126 Sets & Statistics Guide Solutions

s
(x1 x)2 + (x2 x)2 + (x3 x)2 + .....(xn x)2
47. We know that SD =
n

Since # of elements are two, we can reduce the formula to:

s
(x1 x)2 + (x2 x)2
SD =
2

Since the # of elements to be swapped also two, we can deduce;

SD [(x1 x)2 + (x2 x)2 ]

To get a lesser value of SD, the value of [(x1 x)2 + (x2 x)2 ] must be decreased.

Let us calculate the value of [(x1 x)2 + (x2 x)2 ] for the elements to be swapped: {3,
5}, given that x = 5.

For {3, 5}, [(x1 x)2 + (x2 x)2 ] = [(3 5)2 + (5 5)2 ] = 4

The option that has the least value of [(x1 x)2 + (x2 x)2 ] would be the answer as it
would decrease the SD most!

It is to be noted that the sum of the swapped elements {3, 5}, and the elements to be
swapped (all optionsoptions A through E) is 8; this implies that the mean of the set
will not get affected, it will remain 5very important aspect.

Let us take each option and calculate the values.

(A) {0, 8}: [(0 5)2 + (8 5)2 ] = 34 > 4 (derived value); it would rather increase the SD.

(B) {5, 3}: [(5 5)2 + (3 5)2 ] = 4 = 4 (derived value); it would NOT decrease the SD.

(C) {2, 6}: [(2 5)2 + (6 5)2 ] = 10 > 4 (derived value); like option A, it would rather
increase the SD.

(D) {4, 4}: [(4 5)2 + (4 5)2 ] = 2 < 4 (derived value); it would decrease the SDcorrect
answer.

(E) {1, 7}: [(1 5)2 + (7 5)2 ] = 20 > 4 (derived value); like option A, & C, it would
rather increase the SD.

The correct answer is option D.

www.manhattanreview.com 19992016 Manhattan Review


Sets & Statistics Guide Solutions 127

48. Let us understand symmetrical distribution of a characteristic in a large population. It


is also called Binomial distribution or Normal distribution.

99.7% of the data are within


3 Standard Deviations (SD) of the mean

95% within
2 Standard Deviations (SD)

68% within
1 Standard Deviation (SD)

Mean 3SD Mean 2SD Mean SD Mean Mean + SD Mean + 2SD Mean + 3SD

As you see in the figure that there is a bell-shaped curve which is symmetrical about
the mean . Within the range of , approximately 68% population lies, within the
range of 2 , approximately 95% population lies, and within the range of 3 ,
approximately 99.7% population lies.

Some results:

(1) Within the range of & + , approximately 68/2 = 34% population lies

(2) Within the range of & , approximately 68/2 = 34% population lies

(3) Within the range of & + 2 , approximately 95/2 = 42.5% population lies

(4) Within the range of & 2 , approximately 95/2 = 42.5% population lies

(5) Within the range of & + 3 , approximately 99.7/2 = 49.85% population lies

(6) Within the range of & 3 , approximately 99.7/2 = 49.85% population lies

(7) Within the range of & , exactly 50% population lies

(8) Within the range of & , exactly 50% population lies

(9) Within the range of 3 & , approximately (100 99.7)/2 = 0.15% population

lies

19992016 Manhattan Review www.manhattanreview.com


128 Sets & Statistics Guide Solutions

(10) Within the range of + 3 & , approximately (100 99.7)/2 = 0.15% population

lies

(11) Within the range of 2 & , approximately (100 95)/2 = 2.5% population lies

(12) Within the range of + 2 & , approximately (100 ? 95)/2 = 2.5% population lies

(13) Within the range of & , approximately (100 68)/2 = 16% population lies

(14) Within the range of + & , approximately (100 ? 68)/2 = 16% population lies

(15) Within the range of 3 & , approximately (100 0.15) = 99.85% population lies

Or, we can also say, 99.85% of the distribution is more than 3

(16) Within the range of + 3 & , approximately (100 0.15) = 99.85% population lies

Or, we can also say, 99.85% of the distribution is less than + 3

(17) Within the range of 2 & , approximately (100 2.5) = 97.5% population lies

Or, we can also say, 97.5% of the distribution is more than 2

(18) Within the range of + 2 & , approximately (100 0.15) = 97.5% population lies

Or, we can also say, 97.5% of the distribution is less than + 2

(19) Within the range of & , approximately (100 16) = 84% population lies

Or, we can also say, 84% of the distribution is more than

(20) Within the range of + & , approximately (100 16) = 84% population lies

Or, we can also say, 84% of the distribution is less than +

For our question, we are interested in the result 20.a, so 97.5% of the distribution is

more than 2 .

The correct answer is option E.

www.manhattanreview.com 19992016 Manhattan Review


Sets & Statistics Guide Solutions 129

5.1.3 Data Interpretation

49. Since there are odd number (11) of years, the median year would be (11 + 1)/2 = 5th
year, and the corresponding number of unemployed would be the median number of
unemployed, when the columns are arranged in ascending order, taking height into
consideration.

It is not wise to read the value of each column and jot down on the scratch pad and
then after tagging them 1, 2, 3, ....., 11 in ascending order, pick up the 5th ranked value a
median; it would be time-consuming.

Better approach would be to visually pick the 5th ranked bar as per the height of the
bars, and then read the corresponding valuemedian.

Looking at the chart, we find that there are as many as 6 bars whose values are more
than 17500 (Y-axis). Among them, the smallest in height (5th ranked column) would be
the answer. However we find that it is difficult to pin-point which of the two columns,
for years 2003 or for 2011 is the smallest. Well there is no need to be very particular
about it as the questions asks for the closest value of median, and does not ask for the
median year.

Read the data of year 2003 and year 2011.

The least count of Y-axis is 2500; we can approximate the values of year 2003 and year
2011 to be 18000. So the median number of unemployed equals 18000.

The correct answer is option D.

50.

19992016 Manhattan Review www.manhattanreview.com


130 Sets & Statistics Guide Solutions

Country Consumption (million kilograms)

J 1080

K 600

L 360

M 330

N 310

Total 2680

Since these countries consume 40% of the worlds coffee consumption, the countries not
shown in the table above consume (100 40) = 60% of the worlds coffee consumption.

Let the worlds coffee consumption be w million kilograms.

Thus, we have:

2680 = 40% of w

2
=> 2680 = w
5
5
=> w = 2680 . . . (i)
2
3
Thus, coffee consumption of the other countries = 60% of w = w million kilograms.
5

We need to determine the value of coffee consumption of country M as a percent of the


coffee consumption of the countries, i.e.
!
Consumption of M
100
Consumption of other countries
330
=  100%
3
w
5
550
= 100%
w

Using (i), we have:

www.manhattanreview.com 19992016 Manhattan Review


Sets & Statistics Guide Solutions 131

Required percent value

550
=  100%
5
2680
2
550 2
= 100%
2680 5
11
= 100%
134
11
= 100%
132
1
= 100%
12

= 8.3%

Note: The calculation of w and the calculation of the consumption of the other countries
have been avoided.

The correct answer is option A.

51.

Brand Brand
X Y

Miles per gal- 40 36


lon

Cost per gal- $2.40 $2.25


lon

Brand X:

Price per gallon = $2.40

Thus, quantity of gasoline obtained for $36

36 3
= =
2.40 0.2

= 15 gallons

19992016 Manhattan Review www.manhattanreview.com


132 Sets & Statistics Guide Solutions

Distance covered per gallon = 40 miles.

Thus, distance covered using 15 gallons = 15 40 = 600 miles.

Brand Y:

Price per gallon = $2.25

Thus, quantity of gasoline obtained for $36

36 36 4 36 4
= = =
2.25 2.25 4 9

= 16 gallons

Distance covered per gallon = 36 miles.

Thus, distance covered using 16 gallons = 16 36 = 576 miles.

Thus, the required difference = 600 576 = 24 miles.

The correct answer is option B.

52.

Sector Net Income (billion Percent change


$), from
second quarter, 2006 first quarter, 2006

Basic materi- 48 20%


als

Energy 70 +40%

Industrial 45 10%

Utilities 120 +300%

Conglomerates 24 +20%

Let the percent change from first quarter to the second quarter be p%

Thus, we have:

www.manhattanreview.com 19992016 Manhattan Review


Sets & Statistics Guide Solutions 133

p
 
(Income in second quarter) = (Income in first quarter) 1
100
(Income in second quarter)
=> (Income in first quarter) =
p
 
1
100
In the relation above, (+) is used if the percent change is positive and () is used if the
percent change is negative.

Thus, based on the above relation, we have:

Sector Net Income Percent Net Income (billion $) in first quar-


(billion $) change from ter
in second first quarter
quarter

48 48
Basic materi- 48 20% = = 60 ( Greatest))
20

0.8
als 1
100
70 70
Energy 70 +40% = = 50
40

1.4
1+
100
45 45
Industrial 45 10% = = 50
10

0.9
1
100
120 120
Utilities 120 +300% = = 30
300

4
1+
100
24 24
Conglomerates 24 +20% = = 20
20

1.2
1+
100

The correct answer is option A.

53. The number of sales calls that the representative made each month was proportional to
the number of days worked in that month.

Thus, we have:

(Number of calls made in a month) =k(Number of days worked in the month)

In the relation above, k is a constant of proportionality.

Thus, we have:

19992016 Manhattan Review www.manhattanreview.com


134 Sets & Statistics Guide Solutions

Month Number of Number of


days worked calls made

June 20 20k

July 17 17k

August 19 19k

Since the total number of calls made is 168, we have:

20k + 17k + 19k = 168

168
=> k = =3
56

Thus, the number of calls made in August

= 19k = 19 3

= 57

The correct answer is option D.

54. The median of a set of data is the value of the middle term (if there are an odd number
of data values) or the average of the two middle terms (if there are an even number of
data values) after arranging the data in ascending or descending order.

Here, the ages are categorized in ascending order.

Number of data values is 161, which is an odd number.

th
161 + 1

Thus, the median would be the middle term, i.e. term, i.e. the 81st term.
2

Counting the number of terms from the beginning, the 81st term falls under the age
category of 20 29 years as shown below:

www.manhattanreview.com 19992016 Manhattan Review


Sets & Statistics Guide Solutions 135

Age cat- Number of Cumulative number of employees


egory (in employees
years)

Less than 20 29 29

20 29 58 29 + 58 = 87 = > 81st term falls in this cate-


gory

30 39 36

40 49 21

50 59 10

60 69 5

70 and 2
above

Note: Since the actual distribution of ages within the category 20 29 years is not
known, hence, we cannot determine whether the median would fall between 20 24
years or 25 29 years.

The correct answer is option A.

55. Revenue of the particular store in 1994


= 2% of Combined revenue in 1994

2
= 8
100

= $0.16 billion

Revenue of the store in 1995


= 2.3% of Combined revenue in 1995
2.3
= 10
100

= $0.23 billion

Thus, percent increase in revenue of the store

19992016 Manhattan Review www.manhattanreview.com


136 Sets & Statistics Guide Solutions

0.23 0.16
= 100%
0.16
0.07
= 100%
0.16
700
= %
16
640
> %
16

= 40%

Thus, the required percent value is greater than 40%.

The correct answer is option E.

56.
1.6

1.4

1.2

1.0

0.8

0.6

0.4

0.2
2001 2002 2003 2004 2005 2006

Exports in 2001 = $60 million

Imports in 2001
= 0.6
Exports in 2001

=> Imports in 2001 = 0.6 Exports in 2001

=> Imports in 2001 = 0.6 60 = $36 million

Since exports increased by $2 million every year, we have:

Exports in 2006 = $ (60 + 5 2) million = $70 million

Imports in 2006
= 0.8
Exports in 2006

=> Imports in 2006 = 0.8 Exports in 2006

www.manhattanreview.com 19992016 Manhattan Review


Sets & Statistics Guide Solutions 137

=> Imports in 2006 = 0.8 70 = $56 million

Thus, percent increase in imports from 2001 to 2006


Imports in 2006 Imports in 2001
= 100
Imports in 2001
56 36
= 100%
36
20
= 100%
36
5
= 100%
9

= 55.5%

The correct answer is option B.

57. Loan Sanctions as a percent of Total Revenue for Bank X

650
= 100%
1600
650
= %
16
640
= %
16

= 40%
(Note: The actual value will be slightly greater than 40% since 650 was reduced to 640
for ease of calculation)
Loan Sanctions as a percent of Total Revenue for Bank Y
1500
= 100%
3500
15
= 100%
35
300
= %
7

= 42.8%

Thus, the required positive difference


= 42.8 40

= 2.8

19992016 Manhattan Review www.manhattanreview.com


138 Sets & Statistics Guide Solutions

The actual answer will be slightly LOWER than 2.8 (since the value of 40 will actually be
slightly greater).

The correct answer is option B.

58. If balloons are purchased in packets of 10:

2000
Number of packets required = = 200
10

Price of each packet = $9

Thus, total price = $ (9 200) = $1800

If balloons are purchased in packets of 1000:

2000
Number of packets required = =2
1000

Price of each packet = $600

Thus, total price = $ (600 2) = $1200

Thus, money saved = $ (1800 1200) = $600

The correct answer is option D.

59.
500
450
400
350
300
250
200
150
100
50
0
Indiana Vegas Michigan Florida

Number of customers in Indiana and Michigan together


= (250 + 300) thousand

= 550 thousand

Thus, 550 thousand represents 20% of the total number of customers of the company

www.manhattanreview.com 19992016 Manhattan Review


Sets & Statistics Guide Solutions 139

=> 20% of total customers of the company = 550 thousand

=> Total customers of the company

100
= 550
20

= 2750 thousand

Number of customers in Florida = 450 thousand

Thus, the required percent

Customers in Florida
= 100
Total customers
450
= 100%
2750
9
= 100%
55
9
= 100%
54
1
= 100%
6

= 16.7%

The correct answer will be slightly lower than 16.7% (since the denominator was reduced
from 55 to 54 for ease of calculation).

The correct answer is option C.

60. Percent increase in Net Profit from 2010 to 2011

(Net Profit in 2011) (Net Profit in 2010)


= 100
(Net Profit in 2011)
150 125
= 100%
125
25
= 100
125

= 20%

Thus, the percent increase in Net Profit from 2011 to 2012 = 20%

19992016 Manhattan Review www.manhattanreview.com


140 Sets & Statistics Guide Solutions

Thus, Net Profit in 2012

= (100 + 20) % of Net Profit in 2011

= 120% of ($150 million)

120
 
=$ 150 million
100

= $180 million

Total income in 2012 is double the Total Income in 2011

=> Total income in 2012 = 2 ($450 million)

= $900 million

Thus, the required ratio


180
=
900
1
=
5

The correct answer is option A.

www.manhattanreview.com 19992016 Manhattan Review


Sets & Statistics Guide Solutions 141

5.2 Data Sufficiency Questions

Data sufficiency questions have five standard options. They are listed below and will not
be repeated for each question.

(A) Statement (1) ALONE is sufficient, but statement (2) ALONE is not sufficient to an-
swer the question asked.
(B) Statement (2) ALONE is sufficient, but statement (1) ALONE is not sufficient to an-
swer the question asked.
(C) both the statements (1) and (2) TOGETHER are sufficient to answer the question
asked, but NEITHER statement ALONE is sufficient to answer the question asked.
(D) EACH statement ALONE is sufficient to answer the question asked.
(E) Statements (1) and (2) TOGETHER are NOT sufficient to answer the question asked,
and additional data specific to the problem are needed.

19992016 Manhattan Review www.manhattanreview.com


142 Sets & Statistics Guide Solutions

5.2.1 Sets

61. From statement 1:

Since set S has 27 distinct odd numbers, any two numbers would add up to an even
number. However, we have no information about the actual values of the numbers in
the set, there may or may not be a pair in set S, making a sum of 102.

Thus, the answer cannot be uniquely determined. Insufficient

From statement 2:

Since the maximum number is less than 100, all the numbers are less than 100.

However, we have no information about the actual values of the numbers in the set,
there may or may not be a pair in set S, making a sum of 102. Insufficient

Thus, from statements 1 and 2 together:

The set has 27 distinct positive odd numbers (integers), all of which are less than 100.

Thus, possible values of the elements in set S could be:

(1) 1, 3, 5, . . . 47, 49, 51, 53 Minimum possible values of the elements in the set
We can see that 49 + 53 = 102 Satisfies

(2) 99, 97, 95, . . . 53, 51, 49, 47 Maximum possible values of the elements in the set
We can see that 49 + 53 = 102 Satisfies

Thus, in either of the two above cases, we see that there is at least one pair of numbers
which add up to 102.

If we try to exclude 49 and 53 in the set, then we would have to include other numbers
in their places.

Say, for example, we have:

1, 3, 5, . . . 47, 49, 51, 53, 55, 57 We see that 47 + 55 = 102

Thus, irrespective of which numbers are selected in the set, there would always be at
least one pair of numbers which would add up to 102. Sufficient

The correct answer is option C.

www.manhattanreview.com 19992016 Manhattan Review


Sets & Statistics Guide Solutions 143

62. According to the information in the problem, we have:

Boys (x) Girls (y) Total

x
Cleared 20% of x =
5
x 4x
Not cleared x =
5 5

Total x y 50

From statement 1:

Boys (x) Girls (y) Total

x 30
Cleared = =6
5 5
4x 4 30
Not cleared = = 24
5 5

Total x = 50 20 = 30 y = 40% of 50 = 20 50

However, we have no information about the number of girls who cleared the test.

Thus, we cannot determine the total number of students who cleared the test.
Insufficient

From statement 2:

Boys (x) Girls (y) Total

x 1 4x 2x
 
Cleared =
5 2 5 5
4x
Not cleared
5

Total x y 50

However, we cannot determine the total number of students who cleared the test.
Insufficient

Thus, from both statements together:

19992016 Manhattan Review www.manhattanreview.com


144 Sets & Statistics Guide Solutions

Boys (x) Girls (y) Total

x 2x 2 30
Cleared =6 = = 12 6+12=18
5 5 5
4x
Not cleared = 24
5

Total x = 30 y = 20 50

The number of students who cleared the test = 18. Sufficient

Alternate approach:

Total students = 50

Number of girls = 40% of 50 = 20

Number of boys = 50 20 = 30

Number of boys who cleared the test = 20% of 30 = 6

Number of boys who did not clear the test = 30 6 = 24

Since the number of boys who did not clear the test is twice the number of girls who
cleared the test, we have:

24
Number of girls who cleared the test = = 12
2

Thus, number of students who cleared the test = 6 + 12 = 18.

The correct answer is option C.

www.manhattanreview.com 19992016 Manhattan Review


Sets & Statistics Guide Solutions 145

5.2.2 Statistics

Questions on Average (Arithmetic Mean)

63. Say the numbers are x, y, & z.

=> x + y + z = 3 120 = 360

From statement 1:

Average of the first two numbers is 110.


=> x + y = 110 = 220
=> z = 360 220 = 140

Since we do not know the values of x, & y, we cannot get the median.

Say, if x = 0, and y = 220, then upon arranging three numbers in an ascending order,
we get: 0, 140, 220, implying median = 140.

However, if x = y = 110, then upon arranging three numbers in an ascending order, we


get: 110, 110, 140 implying median = 110 (No unique answer).
From statement 2:

Average of the last two numbers is 110


=> y + z = 110 = 220
=> x = 360 220 = 140

Since we do not know the values of y, & z, we cannot get the median.

Thus, from statements 1 and 2 together:

From both the statements, we know that x = z = 140.


=> y = 360 140 140 = 80;
=> Numbers are 140, 80, & 140.

Arranging the three numbers in an ascending order, we get 80, 140, 140. The middle-
most number is 140the median.

The correct answer is option C.

64. Since the numbers are consecutive and there is odd number of integers, 7, the middle-
most integer, d, would be the average (arithmetic mean) of these 7 integers.

19992016 Manhattan Review www.manhattanreview.com


146 Sets & Statistics Guide Solutions

We have to find out the value of d.

Say a = x;
=> b = x + 2, c = x + 4, d = x + 6, e = x + 8, f = x + 10, g = x + 12;

From statement 1:

a + e = 34
=> x + (x + 8) = 34
=> x = 13
=> d = x + 6 = 13 + 6 = 19 (average)

Statement 1 is sufficient.

From statement 2:

c + g = 42
=> (x + 4) + (x + 12) = 42
=> 2x = 26
=> x = 13
=> d = x + 6 = 13 + 6 = 19 (average)

Statement 2 is also sufficient.

The correct answer is option D.

65. Say the 7 numbers are a, b, c, d, e, f , & g, and a = 4 being the smallest.

We have to check whether (a + b + c + d + e + f + g)/7 < 12?

or,

Is (a + b + c + d + e + f + g) < 84?

or,

Is (4 + b + c + d + e + f + g) < 84 ?

=> Is (b + c + d + e + f + g) < 80 ? (1)

From statement 1:

www.manhattanreview.com 19992016 Manhattan Review


Sets & Statistics Guide Solutions 147

Say the largest 4 numbers are d, e, f , & g .


=> (d + e + f + g)/4 = 18.25
=> d + e + f + g = 73

By plugging in the value in equation (1), we get,

Is (b + c + 73) < 80 ?
=> Is b + c < 7 ?

b + c < 7 is not possible as the minimum value of b or c could be 4 since it is given that
the smallest number = 4, so b and c each must at least be 4.

We got the unique answer as No, the average (arithmetic mean) of seven numbers is NOT
less than 12unique answer.

Statement 1 is sufficient.

From statement 2:

Say the smallest 3 numbers are a, b, & c.


=> (a + b + c)/3 = 8.33
=> a + b + c = 25
=> 4 + b + c = 25
=> b + c = 21

By plugging in the value in equation (1), we get,

Is (21 + d + e + f + g) < 80 ?
=> Is (d + e + f + g) < 59?

With b+c = 21, and to keep the values of d, e, f , g to minimum, we can deduce that the
minimum values of b and c each be 10.5, thus each of d, e, f , & g would be greater
than equal to 10.5.

=> Minimum value of (d + e + f + g) = 10.5 4 = 42 < 59 (Answer is YES);

The answer should have been NO as the statement 1, which itself is sufficient, answers
the question as NO. Thus statement 2 must NOT be sufficient.

It is very important to note that in DS problems, if each statement is sufficient to answer


the question, each should render the same answer. In case of Yes/No type of questions,
the answers should be either Yes & Yes or No & No; whereas in case of What is the
value? type of questions, the answers should be exactly the same from each statement.

19992016 Manhattan Review www.manhattanreview.com


148 Sets & Statistics Guide Solutions

However, for sake of understanding, we can analyze it further.

The maximum value of b or c would be (21 4 = 17; as 4 is the smallest number), thus
each of each of d, e, f , & g would be greater than equal to 17 or

=> Minimum value of (d+e +f +g) = 174 = 68 59 (Answer is No; no unique answer);

Statement 2 is not sufficient.

The correct answer is option A.

66. Given that,

Average of 5 numbers = 100;


=> Sum of 5 numbers = 5 100 = 500

Key point to note here is that 5 numbers are real numbers, and not necessarily integers.

From statement 1:

Given that,

None of the five numbers is less than 99.


=> 5 numbers 99; note that a number(s) can be 99.

There can be multiple sets, having average as 100 such as

1. {99, 100, 100, 100, 101} => 3 numbers are equal to 100;
2. {99, 99, 100, 101, 101} => only 1 number is equal to 100; (No unique answer)

Statement 1 is insufficient.

From statement 2:

Given that,

None of the five numbers is greater than 101.

=> 5 numbers 101; note that a number(s) can be 101.

There can be multiple sets, having average as 100 such as

www.manhattanreview.com 19992016 Manhattan Review


Sets & Statistics Guide Solutions 149

1. {99, 100, 100, 100, 101} => 3 numbers are equal to 100;
2. {99, 99, 100, 101, 101} => only 1 number is equal to 100; (No unique answer)

Same sets as discussed in statement 1 are applicable here too.

So, statement 2 is also insufficient.

Thus, from statements 1 and 2 together:

Even combining both the statements cannot get a unique answer as the example sets
discussed above are common to both the statements, thus no unique answer is possible.

The correct answer is option E.

What if the question says? "the five numbers are integers??"

Still the answer would have been E as the example sets discussed above consist of only
the integers, thus no unique answer is possible.

67. The answer would still be E. See how.

From statement 1:

There can be multiple sets, having average as 100 such as

1. {99.5, 100, 100, 100, 100.5} => The question does not say that the numbers are
integers; 3 numbers are equal to 100;

2. {99.5, 99.5, 100, 100.5, 100.5} => only 1 number is equal to 100; (No unique answer)

Statement 1 is insufficient.

From statement 2:

It is clear that statement 2 is also insufficient as the same sets as discussed in statement
1 are applicable here too.

Thus, from statements 1 and 2 together:

Even combining both the statements cannot get a unique answer as the example sets
discussed above are common to both the statements, thus no unique answer is possible.

19992016 Manhattan Review www.manhattanreview.com


150 Sets & Statistics Guide Solutions

The correct answer is option E..

68. The answer would have been D.

From statement 1:

Only possible set to make sure that the average is 100 is {100, 100, 100, 100, 100} as all
the numbers must be integers and greater than 99. None can be 101 or greater as by
doing so, the average, equaling 100 cannot be complied.

Statement 1 is sufficient.

From statement 2:

Similarly, possible set to make sure that the average is 100 is {100, 100, 100, 100, 100}
as all the numbers must be integers and less than 101. None can be 99 or less as by
doing so, the average, equaling 100 cannot be complied.

Thus, statement 2 is also sufficient.

www.manhattanreview.com 19992016 Manhattan Review


Sets & Statistics Guide Solutions 151

Questions on Median

69. The question asks us to compare the average (mean) and the median of set X.

From statement 1:

It states that Set X consists of consecutive multiples of 3.

Note that for any evenly spaced set, Mean = Median

For example, for set: {3, 6, 9, 12, 15}; Mean = Median = 9; and

For set: {3, 6, 9, 12, 15,18}; Mean = Median = (9+12)/2 = 10.5.

The answer is NO, a unique answer!

Therefore statement 1 alone is sufficient.

From statement 2:

It states that the sum of all the terms in set X is 84, but we dont know any information
about the individual elements of the set. We cannot deduce either mean or median.

Therefore statement 2 alone is insufficient.

The correct answer is option A.

70. From statement 1:

These kinds of questions can be solved conceptually.

If we add equal number of terms to the left and the right of the median value, the
median remains unaffected. So for each of the sets P, Q and R, it is insignificant how
many number of terms each set has till we know its median value. The median, 42,
would always lie in the middle, thus the median of the combined set X would also be 42.

Statement 1 is sufficient.

Take another approach...

Say, set P: 40, 42, 50, set Q: {42, 40, 44, 60}, and set R: {8, 4, 40, 44, 60, 110}; median
of each set = 42.

19992016 Manhattan Review www.manhattanreview.com


152 Sets & Statistics Guide Solutions

After combining, set X: {8, 4, 40, 40, 40, 42, 44, 44, 50, 60, 110}; there are 11 terms

The middle most term = (n + 1)/2 = (11 + 1)/2 = 6


=> Median = value of 6th term = 42

Imagine that set R were: {10, 8, 4, 40, 44, 60, 110, 110}.

Will it change the median?

No, it wont as the combined set X has two more terms, but one is added to the left of
the median and the other is added to right of it, thus the median is unaffected.

From statement 2:

The statement itself is clearly insufficient as we do not know the value of medians of the
sets.

The correct answer is option A.

71. From statement 1:

The statement is clearly insufficient as both p & q can have many values.

From statement 2:

Since there is an even number of terms in the list, the median of the numbers would be
the average of the two middle-most terms. Since one of the numbers in the list is 10, to
get median equals to 10 [Median = (10+10)/2 = 10], there must be another number equal
to 10at least one between p & q.

=> p 10 & q 10
The statement too is insufficient as if p = 10, q 10 => no unique value of q.

Thus, from statements 1 and 2 together:

Since from statement 1, p q

=> q must be 10, since the largest of the two must be 10. p can have any value from 10
to (we are interested in p).

The correct answer is option C.

www.manhattanreview.com 19992016 Manhattan Review


Sets & Statistics Guide Solutions 153

72. From statement 1:

When the teams are arranged in ascending order, according to the number of players in
each team, the median number of players per team would be the average of # of players
in the team that is ranked 50% and the average of # of players in the team that is ranked
51%.

Since we do not know the average of # of players in the team ranked 50% and 51%, the
statement is insufficient.

From statement 2:

Following the same analysis as above, this statement is also insufficient.

Thus, from statements 1 and 2 together:

Following both the statement, we can deduce that in the band of 40%60%, where the
teams ranked 50% & 51% lie, the average of # of players per team would be EXACTLY 22
as shown in the image below.

22 or more

22
0% 10% 20% 30% 40% 50% 60% 70% 80% 90% 100%

22 or fewer
Thus, together both the statements are sufficient, and the median is 22.

The correct answer is option C.

73. We cannot determine the median now as we do not know how many players per team
would be there in the teams ranked 50% & 51%.

22 or more

0% 10% 20% 30% 40% 50% 60% 70% 80% 90% 100%

22 or fewer
The correct answer is option E.

19992016 Manhattan Review www.manhattanreview.com


154 Sets & Statistics Guide Solutions

74. The scenario is depicted as below.

23 or more

0% 10% 20% 30% 40% 50% 60% 70% 80% 90% 100%

21 or fewer
Following from both the statement, we can deduce that in the band of 40%60%, where
the teams ranked 50% & 51% lie, the average of # of players per team would be EXACTLY
22 as shown in the image below.

The correct answer is option C.

www.manhattanreview.com 19992016 Manhattan Review


Sets & Statistics Guide Solutions 155

Questions on Average and Median

75. Let A, B, C, D be the scores of four students, and assume that A B C D.


=> Total score = 4 80 = 320

From statement 1:

Since C = D = 120
=> C + D = 240;
So, A + B = 320 240 = 80

Therefore,

1. If A 6= 0, then both A and B scored less than 80 marks;


=> The answer is 2.

However,

2. If A = 0, then B = 80, and only A scored less than 80 marks;


=> The answer is 1.

No unique answer!

This statement alone is not sufficient.

The answers would be B/C/E.

From statement 2:

It states that: median score = 80.

For even number of terms in a set, the median is the average of the two middle terms.

=> B + C = 80 2 = 160

This could happen in two cases

1. When B < 80 and C > 80;

=> A & B scored less than 80. The answer is 2.;

19992016 Manhattan Review www.manhattanreview.com


156 Sets & Statistics Guide Solutions

or

2. When B = C = 80;

=> Only A scored less than 80. The answer is 1; no unique answer.

Statement 2 alone is insufficient.

Thus, from statements 1 and 2 together:

Now, we know that C = D = 120; &

Median = 80 = (B + C)/2 => 80 = (B + 120)/2 => B = 40

=> So, A = 320 40 120 120 = 40

=> A and B, two have scored less than 80; unique answer.

The correct answer is option C.

76. Say the numbers are a, b, c, & d.

=> a + b + c + d = 4 90 = 360

From statement 1:

Given that,

Average (arithmetic mean) of the first, second and fourth integers is 90.

=> a + b + d = 3 90 = 270
=> c = 360 270 = 90

Since we do not know the values of other integers, we cannot get the median.

From statement 2:

Given that,

Average (arithmetic mean) of the first, third and fourth integers is 90.

www.manhattanreview.com 19992016 Manhattan Review


Sets & Statistics Guide Solutions 157

=> a + c + d = 3 90 = 270
=> b = 360 270 = 90

Since we do not know the values of other integers, we cannot get the median.

Thus, from statements 1 and 2 together:

From both the statements, we know that b = c = 90.

=> a + d = 360 90 90 = 180

To get the median, we must know the values of a, and d, however we do not know and it
seems that we cannot get the median.

Upon analyzing, we can deduce the unique value of median despite not knowing the
unique values of a, and d.

Given that four numbers are positive integers, thus

=> Minimum value of a or d would be 1, and the maximum value would be 179

=> So the numbers arranged in ascending order would be 1, 90, 90, 179

=> Median = (90+90)/2 = 90

=> If a & d are equal, each would be 90

=> So the numbers arranged in ascending order would be 90, 90, 90, 90

=> Median = (90+90)/2 = 90 (Unique value)

The correct answer is option C.

77. By now we know that this question can only be solved with both the statements taken
together.

Thus, from statements 1 and 2 together:

From the statement 1, we know that b = 360 3 110 = 30; and

From the statement 2, we know that c = 360 3 80 = 120; and

19992016 Manhattan Review www.manhattanreview.com


158 Sets & Statistics Guide Solutions

=> a + d = 360 30 120 = 210

Say a < 30 & = 20, so d = 210 20 = 190;

Thus the four integers, when arranged in ascending order, would be: 20, 30, 120, 190

=> Median = (30 + 120)/2 = 75;

However, say a = d = 105;

Thus the four integers, when arranged in ascending order, would be 30, 105, 105, 120

=> Median = (105 + 105)/2 = 105 (No unique answer)

The correct answer is option E.

78. From statement 1:

If we arrange the numbers (except x) in ascending order, we have: 1, 2, 4, 6, 18, 19, 20, 21

The middle two terms of the above set are 6 and 18.

Since the median value is x, there must be equal number of terms on either side of x.

Thus, x must be a number between 6 and 18, inclusive.

Thus, possible values of x are: 6, 7, 8, 9, 10, 11, 12, 13, 14, 15, 16, 17 or 18.

Thus, the value of x cannot be uniquely determined. Insufficient

From statement 2:

We know that the mean lies between 12 and 13.

Thus, we have:

x + 1 + 2 + 4 + 6 + 18 + 19 + 20 + 21
12 < < 13
9

=> 108 < x + 91 < 117 => 17 < x < 26

Thus, possible values of x are: 18, 19, 20, 21, 22, 23, 24 or 25.

www.manhattanreview.com 19992016 Manhattan Review


Sets & Statistics Guide Solutions 159

Thus, the value of x cannot be uniquely determined. Insufficient

Thus, from both statements together:

The only possible value of x is 18. Sufficient

The correct answer is option C.

79.
Rugby Baseball
% %

Only Rugby Both Only Baseball


% %

Neither
0%
Total = 100%

We need to determine the percent of students who play only Rugby.

From statement 1:

We cannot determine the value of y only knowing that b = 40. Insufficient

From statement 2:

We cannot determine the value of y only knowing that y b = 20. Insufficient

Thus, from both statements together:

y 40 = 20 => y = 60

From the diagram:


Sum of the values of the percent of students who play only Rugby and the percent of
students who play Baseball (not only Baseball) is 100%.

=> Percent of students who play only Rugby = 100 y = 100 60 = 40% Sufficient

The correct answer is option C.

19992016 Manhattan Review www.manhattanreview.com


160 Sets & Statistics Guide Solutions

80. From statement 1:

Number of men = 60% of 40 = 24.

Number of women = 40 24 = 16.

However, we have no information about the percent of men or women who own a
vehicle. Insufficient

From statement 2:

We have no information either about the number of women in the office or about the
percent of men who own vehicles. Insufficient

Thus, from both statements together:

We have no information about the percent of men who own vehicles. Insufficient

The correct answer is option E.

www.manhattanreview.com 19992016 Manhattan Review


Sets & Statistics Guide Solutions 161

Questions on Range

81. A number when is divided by 7, can leave remainders: 0, 1, 2, 3, 4, 5, or 6.

From statement 1:

Since the range of the remainders is 6, the minimum remainder must be 0 and the
maximum remainder must be 6.

We know that the numbers selected from 1 to 100 are all distinct. However, distinct
numbers may leave the same remainder.

For example, we may have the following cases:

(1) Numbers selected are: 7, 14, 21, 28, 35, 42 and 48.
Thus, remainders obtained on dividing by 7 are: 0, 0, 0, 0, 0, 0, and 6.

Thus, sum of the remainders = 6.

(2) Numbers selected are: 7, 13, 19, 25, 31, 37 and 43.
Thus, remainders obtained on dividing by 7 are: 0, 6, 5, 4, 3, 2, and 1.

Thus, sum of the remainders = 21.

Thus, the sum of the remainders cannot be uniquely determined. Insufficient

From statement 2:

Since the numbers are 7 consecutive integers, the remainders would be all the possible
remainders from 0 to 6, i.e. 0, 1, 2, 3, 4, 5 and 6.

For example: Numbers selected are: 14, 15, 16, 17, 18, 19 and 20.

Thus, remainders obtained on dividing by 7 are: 0, 1, 2, 3, 4, 5, and 6.

Thus, sum of the remainders = 21 Sufficient

Note: Choose any 7 consecutive integers; they are always in the format
of 7n + 0, 7n + 1, 7n + 2, . . . ., 7n + 6. So remainders when these integers are
divided by 7 will be 0, 1, 2, 3, 4, 5 and 6.

The correct answer is option B.

19992016 Manhattan Review www.manhattanreview.com


162 Sets & Statistics Guide Solutions

82. Range of scores for a class = Maximum score for that class Minimum score for that class

From statement 1:

Number of students in each class = 25.

For class B, number of distinct integer scores possible from 76 to 100 (since 100 is the
maximum possible score)
= (100 76) + 1
= 25

Since there are 25 students, and 25 possible scores (as no two students obtained same
score), each student obtained an integer score from 76 to 100.

Thus, maximum score for class B = 100.

Since no student in class A can obtain a score greater than 100 as the maximum
score itself is 100, maximum score for class A will always be less than or equal to the
maximum score for class B.

The answer to the question is No. Sufficient

From statement 2:

Only information about the average score of each class cannot be used to determine the
maximum score obtained in any class. Insufficient

The correct answer is option A.

83. From statement 1:

The smallest element is 2.

We do not know anything about the remaining elements; hence this statement alone is
not sufficient.

Correct answer would be B/C/E.

From statement 2:

The Range is 13.

www.manhattanreview.com 19992016 Manhattan Review


Sets & Statistics Guide Solutions 163

Range = Max. value Min. value

We do not know anything the elements of the set, hence this statement alone is not
sufficient.

The answer could be C/E.

Thus, from statements 1 and 2 together:

Smallest element = 2;

Range = 13

Range = Max. value Min. value


=> 13 = Largest element 2
=> Largest element = 15

Largest element = 15. Answer is NO; unique answer. Hence Statement 1 & 2 together is
sufficient.

There is at least one element which is 15. Note that there may be more than one
elements which are exactly 15.

The correct answer is option C.

Had the questions asked: "Is there only one element equal to 15?"The answer would
have still been E. Number of elements equal to 15 cannot be determined as there may be
2, 3 or more number of elements in the set equal to 15.

84. Given that,

The sequence: 31 4x; x 1

The terms of the set can be formed by plugging in x = 1, 2, 3... onwards in 31 4x.

=> Set: {27, 23, 19, .... up to n terms}; this is an arithmetic progression with first term =
27, and common difference = 23 27= 4

To get the median of the set, we must know the number of terms, n, of the set.

From statement 1:

19992016 Manhattan Review www.manhattanreview.com


164 Sets & Statistics Guide Solutions

Sum of n terms = 99;

We know that,
n
Sum of n terms = [2a + (n 1)d]; where a = first term = 27; and d = common
2
difference = 4

n
=> 99 = [2 27 + (n 1) (4)]
2

Looking at the relationship, we may deduce that n is determinable, so we can get the
finite value of median, but beware the above equation is quadratic and it may render
two values of n. If so, we may not have a unique value of median. So it is recommended
that you solve the equation till the end.

=> 99 = n[27 2(n 1)]; canceling 2


=> 99 = n[27 2n + 2];
=> 99 = 2n2 + 29n
=> 2n2 29n + 99 = 0
=> 2n2 18n 11n + 99 = 0
=> 2n(n 9) 11(n 9) = 0
=> (n 9(2n 11) = 0
=> n = 9 or 5.5 (discarded)
=> n = 9 as n can only be a positive integer.

So the set would be {27, 23, 19, 15, 11, 7, 3, 1, 5}; middle-most value is 11Median

Statement 1 is sufficient.

From statement 2:

We know that the Range = Highest Value Lowest Value;

Since the sequence is arranged in decreasing order, so the highest value = 27.

Lowest value would be the nth term of the arithmetic sequence = 2a + (n 1)d

nth term = a + (n 1)d ;

=> 27 + (n 1) (4)
=> 4n + 31

Given that Range = 32

www.manhattanreview.com 19992016 Manhattan Review


Sets & Statistics Guide Solutions 165

=> 32 = Highest Value (1st term) Lowest Value (nth term)


=> 32 = 27 (4n + 31)
=> 5 = 4n 31
=> n=9

So again the set would be {27, 23, 19, 15, 11, 7, 3, 1, 5}; middle-most value is 11
Median

Statement 2 is also sufficient.

The correct answer is option D.

85. Let the five integers be a, b, c, d and e, such that a < b < c < d < e

Since the mean is 8, we have:

a+b+c+d+e
=8
5

=> a + b + c + d + e = 40 . . . (i)

From statement 1:

Since the median is the middle number, we have:

c=8

Thus, a and b are less than 8, while c and d are greater than 8.

However, the value of the smallest term i.e. a cannot be determined. Insufficient

From statement 2:

Since the range is 6, we have:

e a = 6 => e = a + 6

Thus, from (i):

a + b + c + d + (a + 6) = 40

=> b + c + d = 34 2a

19992016 Manhattan Review www.manhattanreview.com


166 Sets & Statistics Guide Solutions

b+c+d 34 2a
=> =
3 3

Since b, c, d are less than e, their average is also less than e

34 2a 34 2a
=> < e => < a + 6 => 5a > 16
3 3

=> a > 3.2

Thus, the minimum integer value of a is 4.

Similarly b, c, d are greater than a, their average is also greater than a

34 2a
=> > a => 5a < 34
3

=> a < 6.8

Thus, the maximum integer value of a is 6.

Thus, possible values of a are 4, 5 or 6.

Let us verify the above scenarios:

a = 4 => e = 10
Thus, from (i), we have: b + c + d = 40 4 10 = 26
26
This implies that the average of b, c, d is = 8.66
3
Since the numbers are distinct, it is not possible to have three distinct numbers less
than 10 having an average greater than 8. Does not satisfy

a = 5 => e = 11
Thus, from (i), we have: b + c + d = 40 5 11 = 24
24
This implies that the average of b, c, d is = 8.
3
Possible values of b, c, d are: 7, 8, 9; or 6, 8, 10, respectively. Satisfies

a = 6 => e = 12
Thus, from (i), we have: b + c + d = 40 6 12 = 22
22
This implies that the average of b, c, d is = 7.33
3
Since the numbers are distinct, it is not possible to have three distinct numbers
greater than 6 having an average less than 8. Does not satisfy

www.manhattanreview.com 19992016 Manhattan Review


Sets & Statistics Guide Solutions 167

Thus, the only possible value of a = 5. Sufficient

The correct answer is option B.

86. From statement 1:

We know that the range of the numbers x, 2, 3, 7, 11 and 16 is x.

Let us look at possible scenarios:

x is neither the minimum nor the maximum: Range = 16 2 = 14


=> x = 14 Satisfies, since 14 is neither the minimum nor the maximum

x is the minimum number: Range = 16 x = x => x = 8 Does not satisfy, since


8 is not the minimum number

x is the maximum number: Range = x 2, which cannot be equal to x Does not


satisfy

Thus, we have: x = 14. Sufficient

From statement 2:

We know that the mean lies between 8 and 9.

Thus, we have:

x + 2 + 3 + 7 + 11 + 16
8< <9
6

=> 48 < x + 39 < 54

=> 9 < x < 15

Thus, the value of x cannot be uniquely determined. Insufficient

The correct answer is option A.

19992016 Manhattan Review www.manhattanreview.com


168 Sets & Statistics Guide Solutions

Questions on Standard deviation

87. We need not calculate SD to answer the question. To calculate SD, we must be sure that
the value of each element in the set is known. However values of X & Y are not known.

The way X & Y are placed in the set, it hints there values are 11, and 7 respectively,
however we must not assume this. X & Y can have any value, even a decimal or a negative
number!

From statement 1:

Statement 1 alone is insufficient. X & Y can have infinite set of integer values.

Correct answer would be B/C/E.

From statement 2:

Statement 2 alone is insufficient. X can have infinite set of real numbers lying between
12 & 10 such as 12, 11.5, 11.02, ...... , 10.8, 10. Similarly, Y can have infinite set of real
numbers lying between 8 & 6 such as 8, 7, 6.5, ...... , 6.

Note that that having all numbers, except X and Y, as integers in the set does not mean
that X and Y both must be integers too.

Correct answer would be C/E.

Thus, from statements 1 and 2 together:

X can have any value among 12, 11, & 10, complying with being an integer and arranged
in descending order. Similarly, Y can have any value among 8, 7, & 6, but we must pay
attention to the question stem that no two elements are equal, so X cannot be 12 or 10
and similarly, Y cannot be 8 or 6. This implies that X = 12 and Y = 7; unique values for X
and Y.

Thought the question did not ask for the values of X & Y, it asked for SD of the set L, we
have already concluded that once we know the values of all the elements, we can get the
unique value of SD, so SD is determinable as unique value.

So, statement 1 & 2 together is sufficient.

The correct answer is option C.

www.manhattanreview.com 19992016 Manhattan Review


Sets & Statistics Guide Solutions 169

Take Away

Read the question stem carefully.

The information: "Set L: {14, 13, 12, X, 10, 9, 8, Y, 6, 5, 4} has no two equal elements in
the set" worked like a third statement.

88. This is a modified version of the previous question. If you have not attempted the
previous question, do it before seeing this one.

It is clear that the statement 1 & 2 alone are insufficient as X & Y can have infinite set of
values.

Thus, from statements 1 and 2 together:

Even statement 1 & 2 together are not sufficient. As discussed in the previous question
that X can have a minimum value as 10 and a maximum values as 12. Similarly, Y can
have a minimum value as 6 and a maximum values as 8.

So, we can deduce that 2 X Y 6.

As per the statement 2, 5 < X Y 6, but again, X & Y can have infinite number of
real values for them complying with this constraint. Note that unlike in the previous
question, X & Y are not integers in this question!

The correct answer is option E.

Take Away

The key to this question is that if the set is arranged in descending order, it does not
mean that X & Y will only take 11 & 7 as their values, respectively, assuming them as
integers. That was the trap laid in the question.

89. This is a modified version of the previous question. If you have not attempted the
previous question, do it before seeing this one.

From statement 1:

Statement 1 alone is insufficient. X can have three values for itself: 12, 11, & 10;
similarly, Y can have three values for itself: 8, 7, & 6.

From statement 2:

19992016 Manhattan Review www.manhattanreview.com


170 Sets & Statistics Guide Solutions

Statement 2 alone is insufficient. X & Y can have infinite set of real values for them.

Thus, from statements 1 and 2 together:

From statement 1, we know that X can have a minimum value as 10 and a maximum
values as 12. Similarly, Y can have a minimum value as 6 and a maximum values as 8.

So, we can deduce that 2 X Y 6.

As per the statement 2, X Y < 3. Note that as per the statement 1, X & Y are integers.
So X Y = 2, or X = 10, and Y = 8. Unique values of X & Y, thus we can calculate the
unique value of SD.

Statement 1 & 2 together is sufficient.

The correct answer is option C.

Take Away

Read the question stem carefully. The information: "the set is arranged in descending"
worked like a third statement.

90. From statement 1:

Statement 1 is not sufficient as we do not know the values of number of defects in the
remaining 8 TV sets.

Correct answer would be B/C/E.

From statement 2:

Statement 2 is insufficient as we do not know the values of number of defects in any of


TV sets.

Thus, from statements 1 and 2 together:

We know that Range = Max. value Min. value

So, from statement 2, we get,

=> Range = 0 = Max. value Min. value


=> Max. value = Min. value

www.manhattanreview.com 19992016 Manhattan Review


Sets & Statistics Guide Solutions 171

=> # of defects in each of the last 9 sets are equal, however we do not know their
numerical values.

From the statement 1, we know that the # of defects in each of the first 4 sets equals 3.
Since the set # 3 is common in both the statements, hence the # of defects in set 3 = 3.

=> # of defects in each of last 9 sets = 3;

=> # of defects in each of 12 sets = 3;

Since # of defects in each sets = 3, hence there is no deviation at all.

=> SD = 0; unique value

The correct answer is option C.

91. If the set is {5, 5, 5, ...} or {10, 10, 10, ...}; SD = 0, as there is no deviation among the
elements.

Whereas, if the set is {5, 10, 15, ...}; SD 6= 0, as there are deviations among the elements.

From statement 1:

The statement implies that the set may be following;

(1) If the set is {5, 5, 5, ...} or {10, 10, 10, ...}; SD = 0, as there is no deviation among the
elements.
(2) If the set is {5, 5, 5, 5, ...} or {10, 10, 10, 10, ...}; SD 6= 0, as there are deviations
among the elements. Answer is NO; no unique answer.
Hence, this statement alone is not sufficient.

The answer would be B/C/E.

From statement 2:

The statement implies that the set consists of only one element with multiple times.

The set would be one among: {5, 5, 5, ...} or {10, 10, 10,...} or {15, 15, 15, ...}. In each case,
there is no deviation among the elements, therefore SD = 0; answer is YES.

Hence the statement 2 alone is sufficient.

19992016 Manhattan Review www.manhattanreview.com


172 Sets & Statistics Guide Solutions

The correct answer is option B.

92. Deviation of a data point in a data set is its difference from average/mean value.

Range takes into account only the highest value and the lowest value.

Range = Highest Value Lowest Value

Standard Deviation (SD) takes into account each data point. It is a single value for the
deviation for the dataset.

There is no need to know the mathematical formula to calculate SD. GMAT will seldom
ask you to calculate SD; however the understanding of concept is important.

Let us see the definition of SD.

Standard Deviation is mean of the squared differences of data from the Mean.

Say a set is {x1 , x2 , x3 , x4 , ....xn }, and the mean is x , then

s
(x1 x)2 + (x2 x)2 + (x3 x)2 + .....(xn x)2
SD =
n

I know the above seems wild, but as I told you need not calculate the values. The formula
given is for the sake of understanding.

The concept of SD is how far the data points are spread w.r.t. mean. This spread is also
called dispersion.

Less scattered are the data points in a set, less is the value of SD and vice-versa.

Though range, mean, and deviation all characterizations of a data set and are correlated,
you should have an idea that their mathematical definitions vary much. Thus, the in-
stinct should tell you it is very hard to conclude on deviations based on the information
of means and ranges, and the correct answer should be E.

Let us discuss couple of examples to prove that even both statements are not sufficient.

From statement 1:

Case 1:

www.manhattanreview.com 19992016 Manhattan Review


Sets & Statistics Guide Solutions 173

Say set X is {2, 2} and set Y is {0, 2}.

Range of set X, RX = 2 (2) = 4 and Range of set B, RY = 0 (2) = 2. RX > RY .

Mean of set X = XX = (2 + 2)/2 = 0, and for set Y = XY = (0 2)/2 = 1.

s s
(2 0)2 + (2 0)2 (0 + 1)2 + (2 + 1)2
SDx = = 2; SDy = = 1;
2 2

SDx (2) > SDy (1). Set X has greater SD. Answer is YES.

Case 2:

Say set X is {2, 0, 0, 0, 0, 0, 0, 2} and set Y is {0, 2}.

Range of set X, RX = 2 (2) = 4 and Range of set Y, RY = 0 (2) = 2. RX > RY .

Mean of set X = XX = 0, and for mean for set Y = XY = 1.

s
(2 0)2 + 6 (0 0)2 + (2 0)2
SDx = =1
8

SDx (1) 6> SDy (1). Answer is NO. No unique answer.

Statement 1 alone is not sufficient.

The correct answer would be B/C/E.

From statement 2:

Above examples discussed in statement 1 qualify for statement 2 too as for both the
cases, mean of set X (0) is greater than mean of set Y (1) and we did not get a unique
answer.

Statement 2 alone is not sufficient. The correct answer would be between C/E.

Thus, from statements 1 and 2 together:

Combining both the statements will not help as the examples discussed in statement 1
qualify for statement 2 too.

The correct answer is option E.

19992016 Manhattan Review www.manhattanreview.com


174 Sets & Statistics Guide Solutions

93. The set may be something like this: {5, 7, 9, 11, 13, ....}. We do not know from which odd
integer, the sets starts and how many elements are there in it.

Note that: If the elements of a set are evenly spaced and the interval is known, we need
not care for the value of the first element. However to get the value of SD, we must know
the value of number of elements in the set.

From statement 1:

It is sufficient as this gives us the value of number of elements in the set. The set may
be like the following:

{5, 7, 9, 11, 13, 15, 17, 19, 21, 23, 25, 27, 29, 31}

or

{9, 11, 13, 15, 17, 19, 21, 23, 25, 27, 29, 31, 33, 35} or something else, it does not matter.

The correct answer would be A/D.

From statement 2:

It is not sufficient as this does not give us the value of number of elements in the set.
The set may be like the following;

{5, 7, 9}; Mean = 7

or

{3, 5, 7, 9, 11}; Mean = 7, but both the sets have different SDs.

The correct answer is option A.

94. These kinds of DS questions are good questions specially when you are sure that the
answer cannot be A, B, or D, as for the the question, information about set A and set B is
given in two statements. Even if you guess, the probability of getting a success is 0.50.

Note that SDs of two sets would be same if they have equal number of elements, and the
elements are equally placed.

Set A would be like {25, 30, 35, 40, ....}, and set B would be like {18, 21, 24, 27, ....}. Since
the deviation among the elements of set A (5) is greater than the deviation among the
elements of set B (3), we can conclude that the SD of set A is greater than the SD of set

www.manhattanreview.com 19992016 Manhattan Review


Sets & Statistics Guide Solutions 175

B, but the answer is E not C as set B may have more number of elements than that in set
A, resulting in greater value of SD for itself.

Note that we cannot assume that both the sets would have the same number of elements.

Had the # of elements been the same, answer would have been C.

The correct answer is option E.

95. From statement 1:

The statement alone cannot help as we do not know the number of defects in each of
the 9 batchesinformation needed to deduce SD.

If each batch has exactly 3 defects, SD = 0, else SD is indeterminable.

From statement 2:

Though we do not know the number of defects in each of the 9 batches, the information
is sufficient to deduce SD.

Stating that each of the 9 batches has the same number of defects implies that there is
no deviation in the number of defects in 9 batches, thus SD = 0.

The correct answer is option B.

96. Set X has 10 different evenly spaced odd integers, starting from 3. Set Y is made up of
any 8 different odd integers, taken from set X. The integers in set Y may or may not be
equally spaced.

From statement 1:

Given that,

Average of the numbers of set Y = Average of the numbers of set X

Average of the numbers of set X can be deduced conceptually.

"If the numbers of a set are evenly spaced, median of the numbers equals average
(arithmetic mean) of the numbers"

19992016 Manhattan Review www.manhattanreview.com


176 Sets & Statistics Guide Solutions

Since the number of terms in the set X is even, 10, the median would be (11+13)/2 = 12;
or average = median = 12.

Alternatively, we can calculate the average as given below.


Average of the numbers of set X = (3+5+7+9+11+13+15+17+19+21)/10 = 12;
=> Average of the numbers of set Y = 12

Since average of the numbers of set Y (8 integers) = Average of the numbers of set X (10
integers), and two integers that are not in set Y, this implies that the average of those
excluded two integers must also be 12, or the sum of those equal to 2 12 = 24.

If we are able choose only one set of two integers, not a part of set Y, and whose sum
equals 24, we can get the unique set Y, else not. Getting the unique set Y implies that we
can get the unique value of SD.

However there are as many as 5 pairs of two-integers whose sum equals 24 (3, 21), (5,
19), (7, 17), (9, 15), & (11, 13)

Each of the following sets qualified to be set Y have different SDs.

1. Set Y: {5, 7, 9, 11, 13, 15, 17, 19}


2. Set Y: {3, 7, 9, 11, 13, 15, 17, 21}
3. Set Y: {3, 5, 9, 11, 13, 15, 19, 21}
4. Set Y: {3, 5, 7, 11, 13, 17, 19, 21}
5. Set Y: {3, 5, 7, 9, 15, 17, 19, 21}

Statement 1 is clearly insufficient.

From statement 2:

As discussed earlier that there are as many as 5 pairs of integers whose sum equals 24?
(3, 21), (5, 19), (7, 17), (9, 15), & (11, 13), implying different SDs for set Y.

Statement 2 is clearly insufficient.

Thus, from statements 1 and 2 together:

Even combining the two statement would not suffice as the two statements in fact render
the same information.

The correct answer is option E.

www.manhattanreview.com 19992016 Manhattan Review


Sets & Statistics Guide Solutions 177

5.2.3 Data Interpretation

97.
Expenditure on Research
700
Expenditure (million dollars)

600
500
400
300
200
100
0
2010 2011 2012 2013 2014 2015

Institute Govenrment

!
Expenditure by institutes
We need to determine the year in which 100 is the
Expenditure by government
highest.

Thus, we need to expenditure data for each of the 5 years.

From statement 1:

We would be able to determine the expenditure made by the institutes in 2011, and
hence the corresponding percent value for 2011 can be determined.

However, the expenditure made by the government in 2014 is not known. Insufficient

From statement 2:

We would be able to determine the expenditure made by the government in 2014, and
hence the corresponding percent value for 2014 can be determined.

However, the expenditure made by the institutes in 2011 is not known. Insufficient

Thus, from both statements together:

We would be able to determine the expenditure made by the institutes in 2011 as well
as that made by the government in 2014, and hence the corresponding percent values
can be determined. Sufficient

Let us see the calculations:

19992016 Manhattan Review www.manhattanreview.com


178 Sets & Statistics Guide Solutions

Expenditure made by the institutes in 2011 = 120% of that in 2010 = 120% of $200 million
= $240 million

Expenditure made by the government in 2015 = 125% of that in 2014

=> 125% of the expenditure in 2014 = $650 million


100
=> Expenditure in 2014 = $650 million
125
= $520 million

Thus, the corresponding percent values are:

200
2010: 100 = 44.4%
450
240
2011: 100 = 43.6%
550
250
2012: 100 = 41.6%
600
300
2013: 100 = 66.6%
450
250
2014: 100 = 48.1%
520
300
2015: 100 = 46.1%
650

Note: Calculations are not expected from you. This is a data sufficiency question. You
just need to check whether the given data is sufficient to answer the question or not.

Thus, the required year is 2013.

The correct answer is option C.

98. From statement 1:

There is no information about the fares in 2002. Insufficient

From statement 2:

Let the air fare in 2001 be $100.

Thus, air fare in 2002 = 120% of $100 = $120

Thus, we can determine the percent increase in revenue from air fare from 2001 to 2002.

www.manhattanreview.com 19992016 Manhattan Review


Sets & Statistics Guide Solutions 179

However, the relative values of bus fare and railway fare with respect to air fare are not
known.

We cannot assume each of air fare, bus fare and railway fare as $100, since that would
imply assuming that each of the three modes has the same average fare. Insufficient

Thus, from both statements together:

We know the fares of each of the three modes of transport in 2001 and in 2002.

Also, the number of passengers availing each of the three modes of transport is known
in 2001 as well as in 2002.

Thus, the revenue from each of the three modes of transport in 2001 and in 2002 can be
determined.

Thus, the percent change in total revenue from 2001 to 2002 can be determined.
Sufficient

The correct answer is option C.

99. We need to determine the relative values of the revenue earned from Securities in 2011
and in 2012.

From statement 1:

Let the total revenue earned in 2011 be $100.

Thus, revenue earned from Securities in 2011 = 5% of $100 = $5.

Thus, total revenue earned in 2012 = $5 25 = $125.

However, the percent distribution of revenue from different sources in 2012 is not
known.

Thus, the relative value of the revenue earned from Securities in 2012 cannot be
determined. Insufficient

From statement 2:

Revenue earned from Securities in 2010 = 10% of $120 million = $12 million.

19992016 Manhattan Review www.manhattanreview.com


180 Sets & Statistics Guide Solutions

Thus, revenue earned from Securities in 2012 = 75% of $12 million = $9 million.

However, the revenue earned from Securities in 2011 is not known. Insufficient

Thus, from both statements together:

Given that total revenue in 2010 is $120 million, we know that the revenue earned from
Securities in 2012 is $9 million.

However, there is no relation given between the revenue earned in 2010 and 2011.

Also, since the percent distribution of revenue from different sources in 2012 is not
known, the total revenue in 2012 cannot be determined even though the revenue from
Securities in 2012 is known.

Thus, the value of revenue from Securities in 2011 cannot be determined. Insufficient

The correct answer is option E.

100. Let the total number of members be 100.

Thus, we have:

India: 55
USA: x
Canada: 5
UK: 3
Others: y = 100 (55 + x + 5 + 3) = (37 x)

Let the number of countries in the region Others be n.

Thus, the number of members from each of the countries in Others

37x
=
n

We need to determine if:

37x
>x
n

=> 37 x > nx

www.manhattanreview.com 19992016 Manhattan Review


Sets & Statistics Guide Solutions 181

=> x (n + 1) < 37 . . . (i)

From statement 1:

We know that n = 6

However, the value of x is not known.

So, the equation (i) above cannot be verified. Insufficient

From statement 2:

We know that x < 10

However, the value of n is not known.

So, the equation (i) above cannot be verified. Insufficient

Thus, from both statements together:

Possible scenarios:

x = 5, n = 6 => x (n + 1) = 35 < 37 Satisfies (i)


x = 9, n = 6 => x (n + 1) = 63 37 Does not satisfy (i)

Thus, there is no unique answer. Insufficient

The correct answer is option E.

19992016 Manhattan Review www.manhattanreview.com


182 Sets & Statistics Guide Solutions

www.manhattanreview.com 19992016 Manhattan Review


Chapter 6

Talk to Us

Have a Question?
Email your questions to info@manhattanreview.com. We will be happy to answer you. Your
questions can be related to a concept, an application of a concept, an explanation of a ques-
tion, a suggestion for an alternate approach, or anything else you wish to ask regarding the
GMAT.

Please mention the page number when quoting from the book.

GMAC Quant Resources


Official Guide: It is the best resource to prepare for the GMAT. It is a complete GMAT
book. It comes with a Diagnostic test, which helps you measure your capability before-
hand. It features Verbal, Quantitative, and Integrated Reasoning questions types. The
book contains an access code to avail GMATPrep Software, Online Question Bank and
Exclusive Video.

GMATPrep Software: If you buy the OG, you get a free online resource from the GMAC
the testmaker. Apart from practice questions and explanation, it also has two genuine
Computer Adaptive tests; you can also buy four additional CATs and few practice ques-
tions upon the payment.

Best of luck!

Happy Learning!

Professor Dr. Joern Meissner


& The Manhattan Review Team

183

You are a unique candidate with unique experience.
We help you to sell your story to the admissions committee.

Manhattan Admissions is an educational consulting firm that


guides academic candidates through the complex process of
applying to the worlds top educational programs. We work
with applicants from around the world to ensure that they
represent their personal advantages and strength well and get
our clients admitted to the worlds best business schools, gra-
duate programs and colleges.

We will guide you through the whole admissions process:

Personal Assessment and School Selection


Definition of your Application Strategy
Help in Structuring your Application Essays
Unlimited Rounds of Improvement
Letter of Recommendation Advice
Interview Preparation and Mock Sessions
Scholarship Consulting

To schedule a free 30-minute consulting and candidacy evalua-


tion session or read more about our services, please visit or call:

www.manhattanadmissions.com +1.212.334.2500

About the Turbocharge Your GMAT Series (6th Edition)
The Turbocharge Your GMAT Series is carefully designed accessible manner. We delve deeply into the content of
to be clear, comprehensive, and content-driven. Long every single testing area and zero in on exactly what you
regarded as the gold standard in GMAT prep worldwide, need to know to raise your score. The full series is
Manhattan Reviews GMAT prep books offer professional comprised of 16 guides that cover concepts in mathemat-
GMAT instruction for dramatic score improvement. Now in ics and grammar from the most basic through the most
its updated 6th edition, the full series is designed to advanced levels, making them a great study resource for
provide GMAT test-takers with complete guidance for all stages of GMAT preparation. Students who work
highly successful outcomes. As many students have through all of our books benefit from a substantial boost
discovered, Manhattan Review's GMAT books break down to their GMAT knowledge and develop a thorough and
the different test sections in a coherent, concise, and strategic approach to taking the GMAT.

Turbocharge Your GMAT Series


About Manhattan Review About the Author


Manhattan Reviews origin can be traced directly back to Professor Dr. Joern Meissner has more than 25 years of
an Ivy League MBA classroom in 1999. While teaching teaching experience at the graduate and undergraduate
advanced quantitative subjects to MBAs at Columbia levels. He is the founder of Manhattan Review, a world-
Business School in New York City, Professor Dr. Joern wide leader in test prep services, and he created the origi-
Meissner developed a reputation for explaining compli- nal lectures for its first GMAT preparation classes. Prof.
cated concepts in an understandable way. Remembering Meissner is a graduate of Columbia Business School in
their own less-than-optimal experiences preparing for New York City, where he received a PhD in Management
the GMAT, Prof. Meissner's students challenged him to Science. He has since served on the faculties of prestigious
assist their friends, who were frustrated with conventional business schools in the United Kingdom and Germany. He
GMAT preparation options. In response, Prof. Meissner is a recognized authority in the areas of supply chain man-
created original lectures that focused on presenting GMAT agement, logistics, and pricing strategy. Prof. Meissner
content in a simplified and intelligible manner, a method thoroughly enjoys his research, but he believes that
vastly different from the voluminous memorization and grasping an idea is only half of the fun. Conveying knowl-
so-called tricks commonly offered by others. The new edge to others is even more fulfilling. This philosophy was
approach immediately proved highly popular with GMAT crucial to the establishment of Manhattan Review, and
students, inspiring the birth of Manhattan Review. remains its most cherished principle.

Since its founding, Manhattan Review has grown into a www.manhattanreview.com


multi-national educational services firm, focusing on info@manhattanreview.com
+1 (212) 316 -2000
GMAT preparation, MBA admissions consulting, and appli- +1 (800) 246-4600
cation advisory services, with thousands of highly satis-
Manhattan Review, 275 Madison Avenue, Suite 1429, New York, NY 10016.
fied students all over the world. The original lectures have
been continuously expanded and updated by the Manhat-
tan Review team, an enthusiastic group of master GMAT
professionals and senior academics. Our team ensures
that Manhattan Review offers the most time-efficient and
cost-effective preparation available for the GMAT. Please
visit www.ManhattanReview.com for further details.

Vous aimerez peut-être aussi